Sie sind auf Seite 1von 173
HOTS (Higher Order Thinking Skills) CBSE Examination NCERT & Previous Y rs-2014 (Solved) rs Question-Answer Syllabus Covered: ISBN: Published By: CBSE, Delhi 978-93-5058-354-8 © Copyright Reserved by the Publishers All Right reserved. No part of this book may be reproduced, stored ina retrieval system, or transmitted, in any form or by any means, without written permission from the publishers. VK Global Publication Pvt. Ltd. Regd. Office: 4323/3, Ansari Road, Darya Ganj, New Delhi- 110002 Ph: 91-11-23250105, 23250106 Fax: 91-11- 23250141 Corporate Office: 15/1, Main Mathura Contents PART-A: . Electrostatics . Current Electricity . Magnetic Effets of Current and Magnetism . Electromagnetic Induction and Alternating Current Electromagnetic Waves Optics . Dual Nature of Matter and Radiation . Atom and Nuclei . Electronic Devices SCMOONOHA RYN = . Communication Systems Value-based Questions Value-based Questions PART-B: 1. CBSE Examination Papers (Delhi)}—2014 (Solved) 2, CBSE Examination Papers (All India)-2014 (Solved) 3. CBSE Examination Papers (Foreign)—2014 (Solved) Model Test Papers . Model Test Paper 1 (Salved) . Model Test Paper 2 (Solved) . Model Test Paper 3 (Solved) . Model Test Paper 4 (Unsolved) . Model Test Paper 5 (Unsolved) . Model Test Paper 6 (Unsolved) . Model Test Paper 7 (Unsolved) . Model Test Paper 8 (Unsolved) 9, Model Test Paper 9 (Unsolved) 10. Model Test Paper 10 (Unsolved) 11. Model Test Paper 11 (Unsolved) 12. Model Test Paper 12 (Unsolved) 13. Model Test Paper 13 (Unsolved) ONOHEWOND = CENTRAL BOARD OF SECONDARY EDUCATION, NEW DELHI PHYSICS (Theory) CLASS - XIl ‘One Paper Time : 3 hours Marks : 70 Unit No. of Marks. Periods | | Electrostatics 22 15 Il | Current Electricity 20 Magnetic Effect of Current and Ml 22 Magnetism 16 Iv Electromagnetic Induction and 20 Alternating Current V_ | Electromagnetic Waves 04 r T 17 VI | Optics 25 VII | Dual Nature of Matter 08 10 VIIL_ | Atoms and Nuclei 14 IX | Electronic Devices 15 X | Communication Systems 10 " Total 160 70 Unit-!:Electrostatics (Periods 22) Electric Charges; Conservation of charge, Coulomb's law- force between two point charges, forces between multiple charges; superposition principle and continuous charge distribution Electric field, electric field due to a point charge, electric field lines; electric dipole, electric field due to a dipole, torque on a dipole in uniform electric field. Electric flux, statement of Gauss's theorem and its applications to find field due to infinitely long straight wire, uniformly charged infinite plane sheet and uniformly charged thin spherical shell (field inside and outside). Electric potential, potential difference, electric potential due to a point charge, a dipole and system of charges; equipotential surfaces, electrical potential energy of a system of two point charges and of electric dipole in an electrostatic field. Conductors and insulators, free charges and bound charges inside a conductor. Dielectrics and electric polarisation, capacitors and capacitance, combination of capacitors in series and in parallel, capacitance of a parallel plate capacitor with and without dielectric medium between the plates, energy stored in a capacitor. Unit-II:Current Electricity (Periods 20) Electric current, flow of electric charges in a metallic conductor, drift velocity, mobility and their relation with electric current; Ohm's law, electrical resistance, V-| characteristics (linear and non-linear), electrical energy and power, electrical resistivity and conductivity. Carbon resistors, colour code for carbon resistors; series and parallel combinations of resistors; temperature dependence of resistance. Internal resistance of a cell, potential difference and emf of a cell, combination of cells in series and in parallel, Kirchhoff's laws and simple applications, Wheatstone bridge, metre bridge. Potentiometer - principle and its applications to measure potential difference and for comparing EMF of two cells; measurement of internal resistance of a cell. Unit-IIl: Magnetic Effects of Current and Magnetism (Periods 22) Concept of magnetic field, Oersted's experiment. Biot-Savart law and its application to current carrying circular loop Ampere's law and its applications to infinitely long straight wire. Straight and toroidal solenoids, force on a moving charge in uniform magnetic and electric fields, Cyclotron Force on a current-carrying conductor in a uniform magnetic field, Force between two parallel current-carrying conductors-definition of ampere, torque experienced by a current loop in uniform magnetic field; moving coil galvanometer-its current sensitivity and conversion to ammeter and voltmeter. Current loop as a magnetic dipole and its magnetic dipole moment. Magnetic dipole moment of a revolving electron, magnetic field intensity due to a magnetic dipole (bar magnet) along its axis and perpendicular to its axis. Torque on a magnetic dipole (bar magnet) in a uniform magnetic field; bar magnet as an equivalent solenoid, magnetic field lines; earth's magnetic field and magnetic elements. Para-, dia- and ferro-magnetic substances, with examples. Electromagnets and factors affecting their strengths, permanent magnets. Unit-lV:Electromagnetic Induction and Alternating Currents . (Periods 20) Electromagnetic induction; Faraday's laws, induced EMF and current; Lenz's Law, Eddy currents Self and mutual induction. Alternating currents, peak and RMS value of alternating current/voltage; reactance and impedance; LC oscillations (qualitative treatment only), LCR series circuit, resonance; power in AC circuits, wattless current. AC generator and transformer. Unit-V: Electromagnetic Waves (Periods 4) Need for displacement current, Electromagnetic waves, their characteristics, their Transverse nature (qualitative ideas only). Electromagnetic spectrum (radio waves, microwaves, infrared, visible, ultraviolet, X-rays, gamma rays) including elementary facts about their uses. Unit-VI:Optics (Periods 25) Ray Optics: Reflection of light, spherical mirrors, mirror formula, refraction of light, total internal reflection and its applications, optical fibres, refraction at spherical surfaces, lenses, thin lens formula, lens maker's formula, magnification, power of a lens, combination of thin lenses in contact, combination of a lens and a mirror, refraction and dispersion of light through a prism Scattering of light— blue colour of sky and reddish appearance of the sun at sunrise and sunset. Optical instruments: Microscopes and astronomical telescopes (reflecting and refracting) and their magnifying powers. Wave optics: Wave front and Huygens’ principle, reflection and refraction of plane wave at a plane surface using wave fronts. Proof of laws of reflection and refraction using Huygens’ principle. Interference, Young's double slit experiment and expression for fringe width, coherent sources and sustained interference of light, diffraction due to a single slit, width of central maximum, resolving power of microscope and astronomical telescope, polarisation, plane polarised light, Brewster's law, uses of plane polarised light and Polaroids. Unit-Vil:Dual Nature of Matter and Radiation (Periods 8) Dual nature of radiation, Photoelectric effect, Hertz and Lenard's observations; Einstein's photoelectric equation- particle nature of light. Matter waves—wave nature of particles, de-Broglie relation, Davisson-Germer experiment. (experimental details should be omitted; only conclusion should be explained). Unit-Vill:Atoms and Nuclei (Periods 14) Alpha-particle scattering experiment; Rutherford's model of atom; Bohr model, energy levels, hydrogen spectrum. Composition and size of nucleus, Radioactivity-alpha, beta and gamma particles/rays and their properties; radioactive decay law. Mass-energy relation, mass defect; binding energy per nucleon and its variation with mass number; nuclear fission, nuclear fusion. Unit-XI:Electronic Devices (Periods 15) Energy bands in solids (qualitative ideas only) conductors, insulators and semiconductors; semiconductor diode — |-V characteristics in forward and reverse bias, diode as a rectifier; |-V characteristics of LED, photodiode, solar cell, and Zener diode; Zener diode as a voltage regulator. Junction transistor, transistor action, characteristics of a transistor; transistor as an amplifier (common emitter configuration) and oscillator. Logic gates (OR, AND, NOT, NAND and NOR). Transistor as a switch Unit-X:Communication Systems (Periods 10) Elements of a communication system (block diagram only); bandwidth of signals (speech, TV and digital data); bandwidth of transmission medium. Propagation of electromagnetic waves in the atmosphere, sky and space wave propagation, satellite communication. Need for modulation, amplitude modulation and frequency modulation, advantages of frequency modulation over amplitude modulation. Basic ideas about internet, mobile telephony and global positioning system (GPS). = Basic Concepts and Formulae = NCERT Question-Answer = Previous Years' Question-Answer = Important Question—Answer = HOTS (Higher Order Thinking Skills) = Proficiency Exercise = Value-based Questions BASIC Concepts 1. Electric Charges When two bodies are rubbed together, they get oppositely charged Experimental observations show that there are two types of charges: 1. Positive Charge: Positive charge is produced by the removal of electrons from a neutral body. That is, positive charge means deficiency of electrons. 2. Negative Charge: Negative charge is produced by giving electrons to a neutral body. That is, negative charge means excess of electrons on a neutral body. S.1. unit of charge is coulomb (C). 2. Properties of Charges 1. Conservation of Charge: The charge of an isolated system remains constant. This means that charge can neither be created nor destroyed; but it may simply be transferred from one body to another. That is, for an isolated system of bodies 3g = 0. 2. Additive Property: Total charge on an isolated system is equal to the algebraic sum of charges on individual bodies of system. This is called additive property of charges. If a system contains three charges, g;.q2 ,— q3 then total charge on system, ( = q) +42 ~43- 3. Quantisation of Charge: The total charge on a body is intearal multinle of fundamental charce '-' e.g = tne where n is aninteger(n =/, 2, 3,...). 4, Charge is unaffected by motion: The charge on a body remains unaffected of its velocity i, e., charge at rest = charge in motion. 5. Like charges repel while unlike charges attract each other. ‘Coulomb's Law It states that the force of attraction or repulsion between two point charges is directly proportional to the product of magnitude of charges and inversely proportional to the square of distance between them. The direction of this force is along the line joining the two charges, i. v., G19q2 I where k = is constant of proportionality; ¢ is permittivity of fie medium between the charges. If €9 is permittivity of free space and K the dielectric constant of medium, then « = Key I 4192 ~ 4regK 42 For free space K =/, Therefore i 142 F= deg 92 Dielectric constant K :The dielectric constant of a medium is defined as the ratio of permittivity of medium to the permittivity of free space, i.e., K = ¢/ eg Definition of coulomb: 1 coulomb charge is the charge which when placed at a distance of 1 metre from an equal and similar charge in vacuum (or air) will repel it with a force of 9 *109 N. ‘Coulomb's Law in Vector Form Consider two like charges g; and q> located at points A and B in vacuum. The separation between the charges is r. As charges are like, they repel each other, Let Fz; be the force exerted on charge q2 by charge q; and Fie that exerted on charge q, by charge qo . If Tay is the position vector of qo relative tog) and # 91 is unit vector along A to B, then the force Foy is along A to B and A toy B Fox * dey 52 a) Bul F 97= 721 r . Fas 1 9192 Foy _ 4 42 OO ERE firey 2 , Amey 73 T21 Similarly if THe is position vector of q; relative to g> and * 42 is unit vector from B to A, then : 1 4192 aw i qig2 = *5, = “Poy © Arey 72 "21 trey pF Py, (iid = = Obviously 142=- r1, therefore equation (2) becomes i grg2 Fio= Te = (iti) deg 92 121 Comparing (1) and (iii), we get For = Fi2 This means that the Coulomb's force exerted on qx by q) is equal and opposite to the Coulomb's force exerted on q; by qo ; in accordance with Newton's third law, Thus, Newton's third law also holds good for electrical forces. Principle of Superposition ‘Coulomb's law gives the force between two point charges. But if there are a number of interacting charges, then the force on a particular charge may be found by the principle of superposition It States that : If the system contains a number of interacting charges, then the force on a given charge is equal io the vector sum of the forces exerted on it by all remaining charges. The force between any two charges is not affected by the presence of other charges. y Suppose that a system of charges contains n charges. hs 42> 92 ee FO Se O respectively. A paint charge gq is located at Phaving position vector T'relative to O. The total force on q due to all n charges is to be found. If Fi, Fo, Fo, ce Fh are the forces acting on q due to charges qj . 42. 43, --.- Gy Tespectively, then by the principle of superposition, the net force on g is Fe Fs Fot Fatt If the force exerted due to charge q; on q is F,, then from ‘Coulomb's law in vector form I 99 oe = Pe yi F trey | Pada POP) The total force ong due to all x charges may be expressed as > n > n 1 44; » » Here = represents the vector-sum. Continuous Charge Distribution The electrostatic force due to a charge element dg at charge ¢g situated at point P is dot, d 1 gotly 1 109 (7 ~ 5 ~ BR Ft) F The total force on qo by the charged body is j dg(r - qo For linear charge distribution dq = Ad! , where 4 is charge per unit length and integration is over the whole length of charge Y. x For surface charge distribution dy = ods, where ois charge per unit area and integration is for the whole surface of charge. For volume charge distribution dq =p dV’, where p is charge per unit volume and integration is for whole volume of charge. Electric field The electric field strength ai any point in an electric field is a vector quantity whose magnitude is equal to the force acting on per unit Positive test charve and whose direction is along the direction of force. If Fis the force acting on infinitesimal positive test charge F 90 definition, electric field can be given as qq . then electric field strength, E=- .Therefore from » F E = lim — a9 do The unit of electric field strength is mewton/coulomb or volt/metre (abbreviated as N/C or V/m respectively.) 1. The electric field strength due to a point charge ¢ at a distance r in magnitude form \F| I 4 \El=qa = 4iv8y 7? . a in vector form, B= g55, 5 F 2. The electric field strength due to a system of discrete charge is > a e-! : 4, 4nég i=1 3! 3. The electric field strength due to a continuous charge distribution is lg > jae 3 Electric field lines The electric field line in an electric field is an imaginary smooth curve along which an isolated free positive test charge tends to move. In terms of electric field lines the electric field strength is defined as. follows : The electric field strength ai any point is defined as a vector quantity whose magnitude is measured by the number of electric field lines passing normally through per unit small area around that point and whose direction is along the tangent on field line drawn on that point. Accordingly nearer the electric field lines, stronger is the electric field, and farther the electric field lines, weaker is the electric field. In figure, the electric field strength at A is greater than that at B Properties of Electric field Lines 7. to terminate at negative charge. 2. The tangent drawn at any point on the field line gives the direction of electric field strength at that point and the direction of force acting on a positive charge at that point. 3. No two electric field lines can intersect each other because if they do so, then two tangents can be drawn at the point of intersection; which would mean two directions of electric field strength at one point and that is impossible. 4, The electric field lines have a tendency to contract lengthwise like a stretched elastic string and separate from each other laterally. The reason is that opposite charges attract and similar charges repel. . The electric field lines do not form any closed loops. . The equidistant electric field lines represent uniform electric field while electric field lines at different separations represent non-uniform electric field (Figure), __ On =—_ , a aw (2) Uniform electric (b) Non-uniform field electric field Electric Dipole A-system containing two equal and opposite charges separated by a finite distance is called an electric dipole. Dipole moment of electric dipole having charges + q and - q at separation 2/ is defined as the product of magnitude of one of the charges and shortest distance between them. Itis a vector quantity, directed from - 4 to + q. (Remark: Net charge on an electric dipole is zero] Electric Field Due to a Short Dipole 1. A point P on axis, i 2. B= trey 73 2. Ata point P' on equatorial line lp me Ameg PF Electric Force and Torque on an Electric Dipole in a Uniform Electric Field In a uniform electric field of strength £, the net electric force is zero; but a torque equal to »F sin 6 acts on the dipole (where is the angle between directions of dipole moment rf and electric field E). This torque tends to align the dipole along the direction of electric field. Torque in vector form = px E 10. 11. 12. Electric Flux AS cos 6 The total number of electric field lines crossing (or diverging) a surface normally is called electric flux. Electric flux through surface elementA Sis Ap = EA B= EAS cos 0, where Eis electric field strength. Electric flux through entire closed surface is. v=Ts Ed 3s S.I. unit of electric flux is volt metre N.w2e~! Gauss's Theorem It states that the total electric flux through a closed surface is equal to 1 /€g times the net charge enclosed by the surface ie, £9 Formulae for Electric Field Strength Calculated from Gauss's Theorem 1. Electric field due to infinitely long straight wire of charge per unit length A at a distance r from the wire is i 2a E£= Aen, > 13. 2. Electric field strength due to an infinite plane sheet of charge per unit area o is ~— © , independent of distance of point from “Deg the sheel. 3. Electric field strength due to a uniformly charged thin spherical shell or conducting sphere of radius R having total charge q, at a distance r from centre is q (ijal external point By.) = trig Ro (7 > R) (ii)at Surface point Ey = 4 Saray Ro (iiijad internal point Ej, = 0 4. Electric field strength due to a uniformly charged non conducting solid sphere of radius R at a distance r from centre (iyat external point Eyy; = 1 eR iad € h et = Arey ry 0 Ht) (ii)at Surface point Es = i war point Es= on Ro (iiijat internal point Ejny = (r < R) yr dey R3 Electric Potential The electric potential is the physical quantity which determines the direction of charge flow between two bodies when brought in contact. The positive charge always flows from a body at higher potential to that at lower potential. Definition: The electric potential at any point in an electric field is defined as the work done in bringing a unit positive test charge from infinity to that point without acceleration. If W is the work done in bringing infinitesimal positive test charge gq demma imtimins on nlm maint tearm slanician natential pe qo Electric potential at any point is also defined as the negative line integral of electric field from infinity to given point (independent of path followed). ie, o=-[' E.dl The unit of electric potential is volt and its dimensional formula is [ML2T ~3 A- 1], 14. Potential Difference The potential difference between two points in an electric field is defined as the work done in bringing unit positive charge from one point to another. 15. Formulae for Electric Potential 1. due to a point charge g ata point distant r is p= l-9 dey ¥ 2. due to a short electric dipole at a distance r from its centre i. at its axis is i q ve vi firey p? ii. at its equatorial position is zero iii. at a general point having polar coordinates (r, 6) with respect to centre of dipole is. | i peosO > deg y 3. due to a system of charges is 16, 17. 18. VeV 4) t+ Vy Equipotential Surface An equipotential surface is the surface having the same potential at each point. The surface of a charged conductor in equilibrium is a equipotential surface. Electric Potential Energy of a System of Point Charges If, and qo are point charges at separation r;> , then electric potential energy I 4qif2 4aég yl2 If there are n point charges ¢;, q2, -... q, in system at separation rij between i!" and j!# (=1,2,..0 7 =4, 2, ..., 2) then potential energy of system qd zi Gah 2m fF =2 3a) Electric Potential Energy of a Dipole in Uniform Electric Field Potential energy of dipole in uniform electric field is U=-pkcst=- BOE Work done in rotating the dipole in uniform electric field from inclination 8, to 82 W = U2 —U; = pF (cos 84 - cos 82) If dipole is initially in stable equilibrium position (0; = @) and finally its inclination is 6, then 19, 20. 21. 22. W = pE (1 = cos @) Conductors and Insulators ‘Conductors are those substances which contain free charge carriers and so allow easy flow of current. Insulators are those substances which contain practically no free charge carriers and do not allow the flow of current. Free and Bound Charges Inside a Conductor The electrons are free charge carriers inside a metallic conductor while positive ions fixed in lattice are bound charge carriers. Dielectrics and Electric Polarisation The insulators are often referred as dielectrics. Each dielectric is formed of atoms/molecules. In some dielectrics the positive and negative charge centres coincide; such dielectrics are said to be non -polar dielectrics while in some other dielectrics the centres of positive and negative charges do not coincide, such dielectrics have permanent electric dipole moment; such dielectrics are said to be polar dielectrics. The example of polar dielectric is water, while example of non-polar dielectric is carbon dioxide (CO) When a dielectric is placed in an external electric field, the centres of positive and negative dipoles get separated (in non-polar dielectrics) or get farther away (in polar dielectrics), so that molecules of dielectric gain a permanent electric dipole moment; this process is called polarisation and the dipole is said to be polarised. Polarisation is the dipole moment induced per unit volume. Numerically it is equal to surface charge density induced at the faces which are perpendicular to the direction of applied electric field. Capacitor and Capacitance A capacitor contains two oppositely charged metallic conductors at a 23. 24, finite separation. It is a device by which capacity of storing charge may be varied simply by changing separation and/or medium between the conductors The capacitance of a capacitor is defined as the ratio of magnitude of charge (Q) on either plate and potential difference (V) across the plate, i.e., Q Vv The unit of capacitance is coulomb/volt or farad (/’). c= Combination of Capacitors in Series and Parallel 1. Series Combination : When capacitors are connected in series, then net capacitance C is given by fod I i CC) Cot 3 Net charge Q = 4) = 42 = 43 Net potential difference / =V’; + V9 + V3 (remain same) 2. Parallel Combination : When capacitors are connected in parallel, then the net capacitance C=C) + Co + C3 In parallel combination net charge Q=¢1 + 42+ GB Net potential difference V =V;) = Vo = V3 (remain same). ‘Capacitance of Parallel Plate Capacitor A parallel plate capacitor consists of two parallel metallic plates separated by a dielectric. The capacitance of parallel plate capacitor is given by Kegd c= d where K is dielectric constant, A = area of each plate andd = separation between the plates. Special Cases: 1, When there is no medium between the plates, then K = I, so : SA Coacuum = “ = Ca d 2. When space between the plates is partly filled with a medium of thickness ¢ and dielectric constant K, then capacitance c EA £ d=t+ 2 d K Clearly, C > Cg, i.e., on introduction of a dielectric slab between the plates of a parallel plate capacitor, its capacitance increases. 25. Charge induced on a dielectric is - M ) wnere qis free charge on the capacitor plates. 26. Energy stored in a charged capacitor is given by I Qt _ 2a = y U= 9 OF 3D OV joule This energy resides in the medium between the plates. The energy fetared ner init ynliime af a charned ranarifar ic niven hy 3 w= Us! eK? joute fm v2 where £ is electric field strength. NCERT EXERCISES Q.1. Ans. Explain the meaning of the statement "electric charge of a body is quantised". ‘Quantisation of Charge : We know that the charge in a body is produced due to excess or deficiency of electrons. Electron cannot be divided into further smaller parts. Therefore charge on a body is an integral multiple of the charge on electron. The magnitude of charge on an electron is called the fundamental charge or elementary charge. Its value is 1.6 10719 coulomb and is denoted by '’. Therefore, we may say that Any physically existing charge is always an integral multiple of fundamental charge 'e'. This is called the principle of atomicity of charge or principle of quantisation of charge. According to this principle all the continuous values of charge are not possible, but only the integral multiple of fundamental charge 'c' are possible. For example the possible charge on a body may be ¢ , 2¢ , 3e, ... but 0.5¢, 0.77, 2.5¢.... etc. are nat allowed. If ¢ is charge on a body and n is positive or negative integral number, then mathematical form of quantisation of charge is qg=tne where nis integer =1, 2, 3,... ande is a positive quantity equal to + 1.6 x10" 19 coulomb.'e' is also called the quantum af charge. To give 1 coulomb of positive charge to a neutral body, the number of electrons to be removed Q.2. Ans. Ans. Ans. n=q= ! coulomb = 6.25 x 108 e716 % 107! coulomb How can one ignore quantisation of electric charge when dealing with macroscopic or large scale charges. Macroscopic bodies have charges of the order of uC =10" © C. Number of elementary charges on such a body a. 10-6 © 1.6% 197! As charge is created by adding or removing electrons only; therefore addition or removal of a few electrons does not cause any effect on 10'2 electronic charges; hence quantisation of electric charge has no effect at macroscopic or large-scale charges. a= = 6.25 x 10/2 . When a glass rod is rubbed with silk cloth, charges appear on both. A similar phenomenon is observed with many other pairs of bodies. Explain how this observation is consistent with the law of conservation of charge. When a glass rod is rubbed with silk cloth, some electrons of glass rod are transferred to silk, so glass rod acquires positive charge and silk acquires negative charge. Thus, equal but opposite charges appear on both. Initially, before rubbing, the net charge on both glass rod and silk was zero. After rubbing, it is again zero; so law of conservation of charge holds good. . ke? ‘Check the ratio is dimensionless. Look up a table of Gmmy physical constants and determine the value of this ratio. In the ratio ke? Grigmy Fre Unit of constant k= arg? 2 Unit of charge ¢ = coulomb (C) . Fro Nm? Unit of G= 7h = mina ke Unit of me = unit of my = kg ke? Gm,m _ (Nm? /C*)x (©)? = - — = (no unit) (Nm? / kg?) kg? Unitof| P i.g., the ratio is dimensionless. Values of constants k= 9 *109 N-m? /kg?, ¢ =1.6 *10°19 C, G = 6.67 x10°11N ~m2kg? me = 9.1*10731 kg, mp =1.67 «10727 kg kw? (9x 109) x (1.6% 10719)? a Cm, 7 (6-87x10 1) x (9.1 x10 31) x = 2.27x1039 ee (1.67 x 107-27) This is the ratio of electrostatic force and gravitational force between an electron and a proton placed at some distance away. Clearly, electrostatic force is much stronger than the gravitational force. Q. 5. An electrostatic field line is a continuous curve. That is, a field line cannot have sudden breaks. Why not ? Ans. An electrostatic field line is the path of a positive test charge (qq > 0). A moving charge experiences a continuous force in an electrostatic field, so an electrostatic field line is always a continuous curve. Q. 6. Explain why two field lines never cross each other at any paint. Ans. Two electric lines of force can never cross each other because if Ans. intersection (say A); which is impossible. > Ey —O @ E, . The figure below shows tracks of three charged particles 1, 2 and 3 in a uniform electric field E. Give the signs of three charges. Which particle has the highest charge to mass ratio ? 8 {CBSE Dethi 2001] A positively charged particle is deflected towards a negative plate and a negatively charged particle towards a positive plate and shows a parabolic path.. From fig. it is clear that the particles (1) and (2) are deflected towards positive plate; hence, they carry negative charges. Particle (3) is deflected along negative plate, so it carries positive charge. The transverse deflection in a given electric field is 2 gE x y=—at™, where az “and -(2) eZ m u - lf q )Ex” q So y=-| | «4 2\ mJ) y? m From fig., it is obvious that the transverse deflection is the maximum for particle (3), hence, particle (3) has the highest charge to mass ratio (g / m). Q. 8. Which among the curves shown in figure cannot possibly represent electrostatic field lines? Ans. oF . Field lines are wrongly drawn because electric field lines must be normal to the surface of the conductor at each point. . Field lines are wrongly drawn because field lines cannot start from a negative charge. Field lines are correctly drawn, because they are originating from positive charge . Field lines are wrongly drawn as the lines cannot intersect. . Field lines are wrongly drawn because they cannot form closed loops. . Aconductor A with a cavity is given a charge Q.Show that the entire charge must appear on the outer surface of the conductor. Ans. 2. A charged conductor A with a cavity possesses a charge Q. If another conductor B with a charge q is inserted into the cavity, keeping B insulated from A, show that the total charge on the outside surface of A is Q + q. 3. Asensitive instrument is to be shielded from the strong electrostatic fields in its environment. Suggest a possible way. 1. Conductor A has cavity inside it, in which there are no charges. Consider the Gaussian surface 5 lying within the conductor enclosing the cavity. As there is no electric field inside the conductor, the electric flux through the Gaussian surface must be zero, i.¢., This gives charge inside conductor g = 0, i.e., there is no charge inside the conductor and consequently the entire charge remains on the surface of the conductor. 2. When conductor B having charge q is inserted into the cavity, then by induction the charge -7 would appear on inner surface of A and + 4 on the outer surface. Now consider a Gaussian surface 'S' within the conductor enclosing the cavity. Then by Gauss theorem, the net flux through surface S$ will be 1 1 — Eq =—(q-q)=0 feo SE =0 Thus the electric field inside the cavity is zero and so the net charge inside the cavity is zero. Thus the total charge on outer surface of A would be (Q + 9). 3. The electric field inside a metallic case is always zero; so the instrument should be enclosed inside a metallic case. Q. 10.A hollow charged conductor has a tiny hole cut into its surface. Ans. Show that the electric field in the hole is * jf where # is unit 0 vector in the outward normal direction and a is the surface charge density near the hole. Consider a hollow conductor (of any shape) having hole 4 at its surface. Consider a point P near the hole just outside the conductor. First we consider the conductor with hole filled up The electric field at point P due to whole conductor F = « H & This electric field may be considered as the superposition of two electric fields : 1. the electric field due to filled up hole E,. The electric field due to rest of charged conductor Ep. At point P, these electric fields are directed along the same direction so o : foi) € In the interior of conductor electric field is zero. If we consider a point Q near hole but inside the conductor, then electric field Fy changes direction, while electric field E> due to rest of conductor remains the same in magnitude as well as in direction. The net field at Q is Einside = Eg — Ep = 0 PE, = Eg fil) 2. From (i) 2B = To ow Eo 2eq n The electric field strength of the hole will be due ta rest of conductor and is given by Q.11. Ans. Ep= A 2£Q 4. Consider an arbitrary electrostatic field configuration. A small test charge is placed at a null point (é.e., where net electric field E= 0) of the configuration. Show that the equilibrium of the test charge is necessary and unstable. 2. Verify this result for the simple configuration of two charges of the same magnitude and sign placed a certain distance apart. To prove it we suppose that the equilibrium of the test charge e (70) is stable. When the test charge is displaced slightly in any direction, it must experience a restoring force towards the null point. This implies that all the electric field lines near the null point must be directed towards the null point. This means that there is an inward electric flux through a closed surface around the null point. But by Gauss theorem, the electric flux through the surface, not enclosing any charge must be zero. Consequently the equilibrium of a test charge in an arbitrary electrostatic field configuration cannot be stable. Consider the similar charge g, g placed at.4 and B. The mid point C is a null point. If test charge qq is displaced slightly along the line AB, the restoring force produced tends to bring the test charge back at C. But if we displace it normal to the line, then the electrostatic force begins to act so as to take ¢g further away from the null point; so the equilibrium is not stable. For stable equilibrium the restoring force must actin al) directians Q. 12.Two large conducting spheres carrying charges Q4 and Q2 are brought close to each other. Is the magnitude of the electrostatic force between them exactly given by F = 4 Te QQ. 2 , where r is the distance between their centres. Ans. When two charged conducting spheres are brought close to each ‘other, the opposite charges are induced on them and the charge distribution on them is not uniform, So Coulomb's law will not hold good. Hence the force will not exactly be given by the formula. Q. 13.A small test charge is released from rest at a point in an electrostatic field configuration. Will it travel along the field line passing through that point ? Ans. Not necessarily, because the field line, in general, gives the direction of acceleration and not that of velocity. If the field line is a straight line, only then the particle will travel along the field line. Q. 14.1f the Coulomb's law involved 1/73 dependence (instead of 1/ ry, would Gauss's law be still true ? Ans. No, Gauss's law would not apply. Q. 15.What is the work done by a field of nucleus in a complete circular orbit of the electron ? What if the orbit is elliptical ? Ans. Zero; Work done by electrostatic force in a complete cycle is zero, so work done by the field of nucleus in a complete orbit of electron is zero irrespective of the shape of the orbit. Q. 16.We know that electric field is discontinuous across the surface of a charged conductor. Is the electric potential also discontinuous there ? charges will move from higher to lower potential to equalise the potential. Q. 17.Describe schematically equipotential surfaces corresponding to 1. a constant electric field in z-direction. 2. a field that uniformly increases in magnitude but remains in a constant (say z-) direction. 3. asingle positive charge at the origin. 4, auniform consisting of long equally spaced parallel charged wires ina plane. — [CBSE Delhi 2012) Ans. Equipotential surface is surface having the same potential at each of its points. In the given cases the equipotential surface are 1. The planes parallel to XY plane. For same potential difference, the planes are equidistant. y (a) 2. The planes are parallel to YY plane, but for the same potential difference, the separation between the planes decreases. dy>d,>dy {b) 3. Concentric spheres centred at the origin. SZ 4. A periodically varying shape near the grid which gradually attains the shape of planes parallel to grid at far distances. Q. 18.A small sphere of radius 1; and charge q, is enclosed by a spherical shell of radius rz and charge gz . Show that if ¢4 is positive, charge will necessarily flow from the sphere to the shell (when the two are connected by a wire), no matter, what the charge q2 on the shell is. Ans. The potential of inner sphere (due to its own charge and due to charge on shell) is G2 v,-—_[ 2,2) 4zeq\ ry rz } Potential of shell, V) =—L_- 22 7 4mey rp Potential difference, W=V, -V> = toa “ 4t€g ry This is independent of gz . If q1 is positive, the potential of inner sphere is always greater than the potential of shell; so if both inner sphere and shell are connected by a wire, the charge will necessarily flow from sphere to shell. Q. 19.The top of atmosphere is about 400 kV with respect to the surface of the earth, corresponding to an electric field that decreases with altitude. Near the surface of earth, the field is about 100 Vm71. Why then we do not get an electric shock as we step out of our house into the open ? (Assume the house to be steel cage so there is no field inside?) Ans. Our body and ground form an equipotential surface. As we step out into the open, the original equipotential surfaces of apen air change, keeping our head and ground at the same potential; so no charge passes through our body, hence we do not get an electric shock. Q. 20.What meaning would you give to the capacity of a single conductor? Ans. A single conductor is a capacitor with second conductor (plate) at infinity. Q. 21.Guess a possible reason why water has a much greater dielectric constant (= 80) than say, mica (= 6). Ans. The water molecule has a permanent dipole moment, so it has large dielectric strength. Q. 22.4 man fixes outside his house one evening a 2 metre long high insulating slab carrying on its top a large aluminium sheet of area 1 m?. Will he get an electric shock if he touches the metal sheet next morning? Ans. Yes the man will get a shock. The aluminium sheet, insulating slab and ground form a capacitor. The steady discharging current in the atmosphere charges the capacitor gradually to a high voltage and when we touch the metal sheet, we get an electric shock. Q. 23.The discharging current in the atmosphere due to the small conductivity of air is known to be 1800 A on an average over the globe. Why then does the atmosphere not discharge itself completely in due course and become electrically neutral? In other words what keeps the atmosphere charged? Ans. The atmosphere is continually being charged by thunderstorms and lightning all over the glabe and discharged through regions of ordinary weather. On an average the two opposing currents are in equilibrium. Q. 24.What are the forms of energy into which the electrical energy of atmosphere is dissipated during a lightning? Ans. Light energy is involved in lightning, and heat and sound energies in accompanying thunder. Q. 25.4 comb that runs through one's dry hair attract a small bit of paper. Why? What happens if the hair is wet or if itis a rainy day? Ans. When a comb runs through dry hair, it becomes charged due to friction. The charged comb has the ability to attract small bits of paper due to the phenomenon of electrostatic induction. If the hair is wet (or if itis a rainy day), the running of comb on hair is smooth or frictionless; hence comb does not get any charge and hence is not able to attract small bits of paper. Q. 26.Ordinary rubber is an insulator. But the special rubber tyres of aircrafts are made slightly conducting. Why? Ans. On account of friction between tyres and the run—-way, the tyre gets charged. To leak the charge to earth, rubber tyres are made slightly conducting. Q. 27.Vehicles carrying inflammable materials usually have metallic ropes touching the ground during motion. Why? Ans. When a vehicle moves, the body also gets charged due to air -friction along with the tyres. If a vehicle carries inflammable material, the charge accumulated on the body may cause fire. To avoid this, the accumulated charge is conducted to earth through metallic ropes. Q. 28.A sensitive instrument is to be shielded from the strong electrostatic fields in its environment. Suggest a possible way. Ans. There is no electric field or electric lines of force inside a hollow conductor. So, the sensitive instrument may be shielded from the strong electrostatic fields by enclosing it inside a metallic enclosure. Q. 29.4 bird perches on a bare high power line and nothing happens to the bird. A man standing on the ground touches the same Ans. When a bird perches on a bare high power line, its points af contact are at the same potential, so no transfer of charge takes place. In other words no current flows through the bird; so nothing happens to the bird. On the other hand if a man standing on the ground touches. the same line, a potential difference is established between the (touching part) and feet of the man; so there is instantaneous flow of sufficient quantity of charge from the power line to the earth through the conducting human bedy; hence the man gets a fatal shock. IMPORTANT Questions VERY Short Answer Questions 1 Marks Previous Years’ Questions Q. 1. The force acting between two point charges q4 and q2 kept at some distance apart in air is attractive or repulsive when [CBSE (F) 2007) 1. qig2 <0 2. qag2> 0. Ans. 1. repulsive 2. attractive Q. 2. Write S.L unit of = [CBSE Delhi 2003) 1. electric field intensity and 2. electric dipole moment. Ans. 1, $.I. unit of electric field intensity is newton/coulomb (NC~‘). 2. S.|. unit of electric dipole moment is coulomb x metre (C_~) Q. 3. Name the physical quantity whose S.1. unit is JC~1. Is it a scalar oravector quantity? = (CBSE Delhi 2010) Ans. Electric potential. It is a scalar quantity. Ans [CBSE Delhi 2012] . The charge inside the conductor is zero, so electric field is zero. Q. 5. Name the physical quantity which has its unit joule coulomb”? . Ans. Ans. Q.9. Ans Is ita scalar or a vector quantity? = (c8se Delhi 2010) The physical quantity having unit joule coulomb -1 is electric potential. It is a scalar quantity. . Why must electrostatic field be normal to the surface at every point of a charged conductor? — (CBSE Delhi 2012) “In the static situation, E has to ensure that the free charges on the surface do not experience any force. . Why is electrostatic potential constant throughout the volume of the conductor and has the same value (as inside) on its surface? = [CBSE Delhi 2012) . Electric field intensity is zero throughout the volume and the potential just inside, has to be equal to potential on the surface. . What is the geometrical shape of equipotential surfaces due to asingle isolated charge? — [CBSE Delhi 2013) Spherical shape with point charges q > 0 or q <0 at the centre. Depict the equipotential surfaces for a system of two identical positive point charges placed a distance 'd' apart. (CBSE Delhi 2010) . Equipotential surfaces due to two identical charges is shown in fig (Al) 2008] Ans. E — = = =] =| Q. 11.Draw an equipotential surface for a system consisting of two charges ges Q, — Q separated by adistancer inair. (CBSE (Al) 2013, 2008) Ans. Electric potential is zero at all points in the plane passing through the dipole equator. Lf Q. 12.Sketch the electric lines of force for two point charges g4 and 42 for q4 = q2 and q4 > gz separated by a distance d. Ans. When the charges are equ¢/, the neutral point N lies at the cenire of the line joining the charges. However, when the charges are unequal, the point N is closer to the smaller charge What is the angle between the directions of electric field at any 4. axial point and 2. equatorial point due to an electric dipole? Ans. ati . =. . The directions of electric field 4 at axial point P1 and electric field Eat equatorial point Pz are shown in fig. Obviously, angle 2 2 between Ey and Ee is 180° Q. 14.Express dielectric constant in terms of capacitance. [case Delhi 2006, 2005, 2002] Ans. The dielectric constant of a medium Is defined as the ratio of capacitance of capacitor when filled with a medium to capacitor of same capacitance when medium is removed -_ Gmedium ie, K= : Co What is the function of a dielectric in a capacitor ? = [CBSE Delhi 2002] Ans. Dielectric reduces the effective charge on plates and hence increases the capacitance. Q. 16.How does the electric field inside a dielectric decrease when it is placed in an external electric field ? = (CBSE Delhi 2005) Ans. When a dielectric is placed in an external electric field, the charges are induced on the faces of dielectric which produce opposite electric field in the dielectric, thus net electric field is reduced. Q. 17.Define electric dipole moment. Is it a scalar or a vector quantity ? What is its S.I. unit? = (CBSE (Al) 2011, Delhi 2006, (F) 2012, 2009 Ans. The electric dipole moment is defined as the numerical product of either charge and the distance between the charges. Its direction is from negative to positive charge. ie,, pag Electric dipole moment is a vector quantity. Its SI unit is coulomb-metre. Q. 18.An electric dipole of dipole moment 20 x 1076 C is enclosed by closed surface. What is the net electric flux coming out of this surface? [CBSE Delhi 2005) Ans. Zero. Reason : Net charge enclosed by surface = Net charge on dipole = q-q=0 «. Total electric flux = 1 / €9 x net charge enclosed = 0 Q. 19.Fig. shows three point charges +2q, -q and +3¢. Two charges +29 and -q are enclosed within a surface 'S'. What is the electric flux due to this configuration through the surface 'S'? +3g0 o [CBSE Delhi 2010] Ans. Electric flux = 1 / 9 x (Net charge enclosed within the surface) I “2, ) 1 = 4g —g= g eo i~4 eo i Q. 20.Two charges of magnituds - 2Q and + Q are located at points (a, 0) and (4a, 0) respectively. What is the electric flux due to these charges through a sphere of radius '3a' with its centre at the origin? [CBSE (Al) 2013] Ans. = Electric flux, ® = 20 £0 Concept: 1. Mark the position of the charges on number line. 2. Draw a sphere of radius 3a about the origin and observe that which charge is inside the sphere, and then use Gauss theorem. Q. 21.Sketch graph to show how charge Q given to a capacitor of capacitance C varies with the potential difference. Ans. The graph of charge (Q) versus potential difference (I’) is a straight line whose slope is equal to capacitance 'C' oO Slope = tand Vv Q. 22.The graph shown here shows the variation of total energy (Z) stored in a capacitor against the value of the capacitance (C) itself. Which of the two : the charge on capacitor or the potential used to charge it is kept constant for this graph? —m —>c Ans. The given graph represents Bee i This is satisfied by the expression, B= oa for constant ¢ That is, the charge (7) is kept constant. corresponds to its = [CBSE Delhi 2010) 1. stable and 2. unstable equilibrium? Ans. 1. In stable equilibrium the dipole moment is parallel to the direction of electric field (i.2., @ = 0). 2. In unstable equilibrium P.E. is maximum, so @ = 7 so dipole moment is antiparallel to electric field. Q. 24.The force between two point charges kept at a distance r apart in air is F. If the same charges are kept in water at the same distance, how does the force between them change? Ans. The force in air 1 4192 frey 2 a= The force in water i Giga brsgk 2 Fy I Fy K Dielectric constant of water is 81, so the force in water reduces to 1 / 81 times. a Q. 25.A uniform electric field exists between two charged plates as shown in the fig. What should be the work done in moving a charge q along the closed rectangular path ABCDA. Ans. Work done in an electric field is independent of the path and depends only on the initial and final positions. Here initial and final points are coincident. Work = ¢ * (Vyingl —Vinitiat) = 9 (Va —Va ) =O So, net work done is zero. Q. 26.What should be the work done if a point charge + q is taken from a point A to the point B on the circumference drawn with another point + q atthe centre? = [CBSE Delhi 20010) Ans. ge tN - wA 7 . i ‘ ' Oo Hy I ° ' ‘ +q : --B The potential of points A and B are same being equal to , , ig Vy=Vp= 1 UE 400g R where & is the radius of the circle. Work done W = (VB -Va)=¢ (Va -Va) = 0. Q. 27.A point charge Q is placed at point O as shown in the figure. Is the potential difference V, - Vg positive, negative, or zero, if Q is [CBSE Delhi 2074] 1. positive 2, negative? Ans. The potential due to a point charge decreases with increase of distance. So, in case 1. V4 —Vp is positive. For case 2. V4 —Vx is negative. Q. 28.Why is the potential inside a hollow spherical charged ‘conductor constant and has the same value as on its surface? [CBSE (F) 2012] Ans. Electric field intensity is zero inside the hollow spherical charged conductor. So, no work is done in moving a test charge inside the conductor and on its surface. Therefore, there is no potential difference between any two points inside or on the surface of the conductor. po V, -Vg =-[ E.dl=0 = V, Vp = Constant Q. 29.Why do the equipotential due to a uniform electric field not intersect each other? — [CBSE (F) 2012] Ans. This is because, at the point of intersection there will be two values of electric potential which is not possible. Q. 30.A hollow metal sphere of radius 5 cm is charged such that the potential on its surface is 10 V. What is the potential at the centre of the sphere? = [CBSE (Al) 2011] Ans. Potential at centre of sphere = 10 V. Potential at all points inside the hollow metal sphere (or any surface) is always equal to the potential at the surface. Ans. +e tt eetst <—_v—> Two protons, A and B, are placed between two parallel plates having a potential difference V as shown in fig. Will the protons experience equal or unequal force ? The electric field strength at all points between the two oppositely charged parallel plates are same; so protons A and B will experience equal force Vv d where d is separation between the plates. Fy =Fp=ek =e Q. 32.How much work is done in moving a 500 pC charge between Ans. two points on an equipotential surface. [CBSE Delhi 2002) OR Why is there no work done in moving a charge from one point to another on an equipotential surface? = [CBSE (F) 2012] For equipotential surface each point is at the same potential, so Veinat =Vinitial so Wes ¢ (Vpinal =Vinitiad) =O OR The potential difference between any two points of equipotential surface is zero. We have “we=0 So, the work done in moving a charge on an equipotential surface is. zero, Q. 33.The distance of the field point, on the equatorial plane of a small electric dipole is halved. By what factor does the electric field due to the dipole change? —[C8sé Delhi 2004C) Ans. For small dipole, ip I = oc Eeguator 3 ; qalor toreg pd aH When r is halved, the electric field strength become 8-times of the original field. Q. 34.The distance of the field point on the axis of a small dipole is doubled. By what factor will the electric field, due to the dipole change? [CBSE Delhi 2004C] Ans. For a small dipole, 1 2p 2 3G trey > r When the distance y is doubled, the electric field strength becomes 4 / 8 times the original field Eaxis = Q. 35.Name the dielectric whose molecules have = [CBSE (Al) 2004¢) 1. non-zero and 2. zero dipole moment. Ans. 1. The dielectric having non-zero dipole moment is water. 2. The dipole having zero dipole moment is diamond (or silicon). Q. 36.Define dielectric constant of a medium. What is the value of dielectric constant for a metal ? = (CBSE Delhi 2004¢) Ame Tho diclactric canctant nf a medinm ic dafinad an the ratin af permittivity of medium to the permittivity of free space. The dielectric constant is also called the relative permittivity of medium e=K=— ey The value of dielectric constant for a metal is infinity #.e.. Kyesat = Q. 37.If the radius of the Gaussian surface enclosing a charge is halved, how does the electric flux through the Gaussian surface change? — [CBSE (Al) 2008C] Ans, Electric flux through a Gaussian surface, enclosing the charge q is I OF = 4 eo # This is independent of radius of Gaussian surface, so if radius is halved, the electric flux through the surface will remain unchanged. Q. 38.A metallic sphere is placed in a uniform electric field as shown in the figure. Which path is followed by electric field lines and why? > ® Ans. Path (d) is followed by electric field line. Reason: There are no electric field lines within a metallic sphere and field lines are normal at each point of the surface. Q. 39.A charge QuC is placed at the centre of a cube. What would be the flux through one face ? = [C&SE (Al) 2012, CBSE (F) 2010) Ans. o Electric flux through whole cube = 2 Electric flux through one face £0 Q. 40.A charge g is placed at the centre of a cube of side I. What is the electric flux passing through two opposite faces of the cube? = [CBSE (Al) 2012] Ans. By symmetry, the flux through each of the six faces of the cube will be same when charge q is placed at its centre Other Important Questions Q. 41.An arbitrary surface encloses a dipole. What is the electric flux through this surface? = (NCERT Exemplar) Ans. Net charge on a dipole = — g + 4 = 0. According to Gauss's theorem, electric flux through the surface, > > peas sx gn0 £9 Q. 42.4 metallic spherical shell has an inner radius R4 and outer radius R2. A charge Q is placed at the centre of the spherical cavity. What will be surface charge density on = (NCERT Exemplar] 41, the inner surface, and 2. the outer surface? Re Ans. +O When a charge + Q is placed at the centre of spherical cavity, the charge induced on the inner surface = ~ () the charge induced on the outer surface = + Q -o 4rR?, +Q fT R? > =". Surface charge density on the inner surface = Surface charge density on the outer surface = Q. 43.The dimensions of an atom are of the order of an Angstrom. Thus there must be large electric fields between the protons and electrons. Why then is the electrostatic field inside a conductor zero? = (NCERT Exemplar] Ans. The electric fields bind the atoms to neutral entity. Fields are caused by excess charges. There can be no excess charge on the inter surface of an isolated conductor. So, the electrostatic field inside a conductor is Zero. Q. 44.Sketch the electric field lines for a uniformly charged hollow cylinder shown in figure. [NCERT Exemplar] tte tee etet Ans. + f+ \ { Zw | | —C)— — b+ FYN Top view Side view You are given three uncharged spherical conductors A, B and C, touching each other. A glass rod carrying positive charge is brought near sphere A as shown. What will be the charges on spheres A, B and C? Ans. By induction, the electrons will be transferred from C to 4, so sphere Awill be negatively charged and C will be positively charged; while sphere # will remain neutral Q. 46.A proton and an electron are placed freely in an electric field. Which of the particles will have greater acceleration and why? Ans. Force on a charged particle in an electric field P= 96 for an electron or proton g = ¢ ) F = cE (same for electron and proton but afpositely directed) If m is mass of particle Foek oe acceleration a = = m we As mass of electron is smaller than that of proton, the acceleration of electron will be greater than that of proton. Q. 47.4 small metallic charged sphere is placed at the centre of large uncharged spherical shell and the two are connected by a wire. Will any charge flow on the outer shell ? Ans. Yes, the whole charge will flow to the outer shell; because charge always resides on outer surface of a conductor. Q. 48.Does the electric potential rise or fall along an electric line of force? Ans. The electric potential falls along a line of force because a line of force is always directed from higher to lower potential. Q. 49.Can electric potential at a point be zero, while the electric field is not zero? Ans. Yes, electric potential is zero at all points on equatorial line of electric dipole, while electric field strength is not zero Q. 50.Can electric field at a point be zero, while electric potential is not zero? Ny tem eetad en om bend) ocgcy be oerec al pummel Moc mmeeedecmbee ble abeambeta Gale te zero, but electric potential is finite. Q. 51.Consider twe conducting spheres of radii Ry and R2 with Ry > Ans. Rp. If the two are at the same potential, the larger sphere has more charge than the smaller sphere. State whether the charge density of the smaller sphere is more or less than that of the larger one. = [NCERT Exemplar] Since two spheres are at the same potential, therefore Vi5V2 Q Qo IreyR, Ftaeghs Q, Ry > — = os Qo Ro 0 Given, Ry, > Ro Qr > Qs = Larger sphere has more charge Q Qe now, oy = 5 and, o2 = 5 4nR)? 4aR2? a Q Ry? o QI Ro? a2 Rp RY Ry > = = [From equation (t)] a Kp Re Ro Since R; > Ro, therefore fore a9 > o; Charge density of smaller sphere is more than that of larger one. Q. 52.Do free electrons travel to region of higher potential or lower Ans. Q. 53.Can there be a potential potential? [NCERT Exemplar] Free electrons would travel to regions of higher potentials as they are negatively charged. erence between two adjacent Ans. Yes, if the sizes are different. Q. 54.Two protons are brought nearer; what will be the effect on potential energy of system? Ans. A repulsive force acts between protons, if they are brought nearer, work must be done by external force; hence the potential energy of system increases. Q. 55.An electron and a proton are brought nearer; how does the potential energy of system change? Ans. There is attractive force between an electron and a proton, therefore when they come nearer, the work is done by the system itself and so the potential energy of system decreases. Q. 56.A charge g is placed inside a closed surface. What is the electric flux through the surface? Ans. From Gauss theorem electric flux, P= q / «y Q. 57.4 charge q is placed just outside a closed surface. What is the electric flux through the surface? Ans. Zero, charge enclosed by surface Xy = 0. Q. 58.There are two metallic spheres; one is bigger than the other. Which has greater capacitance? Ans. The bigger sphere has greater capacitance since capacitance of a spherical conductor C sx +, Q. 59.Where does the energy of a charged capacitor reside? Ans. The energy of a charged capacitor resides in the medium between the plates. Q. 60.Two spheres of copper of the same radii, one hollow and the other solid, are charged to the same potential. Which sphere possesses more charge? Ans. The radii of both the sphere are equal, therefore their capacitances will he the same and hence their charnes 0 =Cl’ will be same. Q. 61.ls there any conductor which can be given any amount of charge? Ans. Yes, earth is such a conductor. Any quantity of charge given to it retains its potential zero. Q. 62.The capacitance of a charged capacitor is C and the energy stored in it is U. What is the value of charge on the capacitor ? Ans. Energy stored U’ = Q / 2C where Q is charge on capacitor, =? = 2cU Charge @ = V20U Q. 63.What is the net charge on a charged capacitor? Ans. Zero; since charges on plates of capacitor are + Q and — Q; ©Q = +Q-Q=0 Q. 64.Can we give any amount of charge to a capacitor? Why? Ans. No. Reason: When we give charge to a capacitor continuously, the potential difference between its plates goes on increasing. Finally a stage will come when the electric field between the plates will exceed the dielectric strength of medium; thus puncturing the dielectric i.e. breaking the dielectric into positive and negative charges; so the capacitor will be discharged giving sparking. Q. 65.Which of the following is a dielectric substance: Germanium, mica, carbon? Ans. Mica. Q. 66.Why should circuit containing capacitor be handled causiously even when there is no current? Ans. When there is no current in the circuit, the capacitor may have charge. Therefore by handling a charged capacitor a person may get a severe shock; hence the circuit containing a capacitor must be handled causiously. Q. 67.s there any material which when inserted between the plates of Ans. a capacitor reduces its capacitance? No, for any material K > J. Q. 68.The plates of a charged capacitor are connected by a voltmeter. Ans. If the plates of the capacitor are moved farther apart, what will be the effect on the reading of voltmeter? Capacitance C & 1 / d,i.e., when plates are moved farther, the capacitance decreases. As charge (() on capacitors is constant, the potential ’ = ( / C increases, hence reading of voltmeter increases. SHORT Answer Questions 2,3 Marks Previous Years’ Questions Q.1. Ans. Q.2. Ans. Two insulated charged copper spheres A and B of identical size have charges g4 and —3q, respectively. When they are brought in contact with each other and then separated, what are the new charges onthem? [CBSE (F) 2011] 94 ~ 344 2 Change on each = = -44 Two insulated charged copper spheres A and B of identical size have charges q4 and gz respectively. A third sphere C of the same size but uncharged is brought in contact with the first and then in contact with the second and finally removed from both. What are the new charges on Aand B? [CBSE (F) 2011] 2 New charge on A is * and New charge on Bis ™ ‘ “18 = 4, . Plot a graph showing the variation of coulomb force (F) versus 1 | = } , where r is the distance between the two charges of each r? Ans. Ans. pair of charges: (1 wC, 2 pC) and (2 uC, - 3 pC). Interpret the graphs obtained. = [CBSE (Al) 2011] i 4ide direg pe Hassing The graph I isa straight line 1 passing or ae . qrq2 through between F and r? of slope 4a) * origin ! Ve— ‘epulsive Attractive F a 4 Since, magnitude of the slope is more for attraction, therefore, attractive force is greater than repulsive force. . Define electric field intensity. Write its S.1. unit. Write the magnitude and direction of electric field intensity due to an electric dipole of length 2¢ at the mid point of the line joining the two charges. CBSE (Al) 2005) Electric Field Intensity : The electric field intensity at any point is defined as the electric force per unit positive test charge when it is placed at that point i.e., Ee E oes lim — | % 9 do a eg fc #4 ate a The §.1. unit of electric field intensity is newton/coulomb. Electric Field Strength at mid point of dipole: The electric field strength at mid point C due to charge + g and — g are along the same direction. i q i q i 24 Esk, + Bo = = i ee ae en, ee Ans. Ans. Its direction is from +q to- 4g. . The electric field E due to any point charge near it is defined as an . . . E bel — where g is the test charge and F is the force acting q>0¢ ‘on it. What is the physical significance of un. in this expression? Draw the electric lines of point charge Qwhen [CBSE Delhi 2007) 1. Q>0and 2.Q<0. The physical significance of os in the definitions of electric field E= lim Ei q>0g The point charge q produces its own electric field, hence it will modify the electric field strength to be measured. The test charge used to measure the electric field must be too small or eae The electric lines of force are shown in fig below. (ha>0 aco . Draw the electric field lines due to a uniformly charged thin spherical shell when charge onthe shellis = (CBSE Delhi 2008) 1. positive and 2. negative. The electric field lines are shown in the fig. For a positively charged shell, the field lines are directed radially in outward direction and for nanatively charged chall thece are directed in radially inward direction. (a) Positively charged shell (a) Negatively charged shell Q. 7. Following figure shows the electric field lines around three point charges A, B and C. INCERT Exemplar] 41. Which charges are positive? 2. Which charge has the largest magnitude? Why? 3. In which region or regions of the picture could the electric field be zero? Justify your answer. near A, near B, near C, iv. nowhere. Ans. 1. Charges A and C are positive since lines of force emanate from them. 2. Charge C has the largest magnitude since maximum number of field lines are associated with it 3. (i) near A. There is no neutral point between a positive and a negative charge. A neutral point may exist between two like charges. From the figure we see that a neutral point exists between charges A and C. Also between two like charges the neutral point is closer to the charge with smaller magnitude. Thus, electric field is zero near charge A. Q. 8. Establish the relation between electric field and potential gradient. Ans. Let us consider two closely spaced equipotential surfaces A and B as shown in figure. Let the potential of A be Vq =V and potential of B be Vg =V - dV where dV is decrease in potential in the direction of electric field E normal to Aand B. Let dr be the perpendicular distance between the two equipotential surfaces. When a unit positive charge is moved along this perpendicular from the surface B to surface A [ Equipotentials against the electric field, the work done in this process is Wea = _Efdr\ Thie werk dane annale the natantial differance \/. _\s. Wea = Va - Vg =V-(V - dV) = dV -. ~Edr= dV dV or, E= — d = negative gradient of potential. ir Q.9. 1. Define electric flux. Write its S.1. unit. 2. A spherical rubber balloon carries a charge that is uniformly distributed over its surface. As the balloon is blown up and increases in size, how does the total electric flux coming out of the surface change? Give reason. (CBSE Delhi 20080, 2007] Ans. For first part refer to point 10 of basic concepts. 4 &) As charge remains unchanged when size of balloon increases, electric flux through the surface remains unchanged. Total electric flux through the surface = Q. 10.State Gauss's law in electrostatic. A cube with each side ‘a' is kept in an electric field given by B=cxt (as is shown in the figure) where C is a positive dimensional constant. Find out [CBSE (F) 2012} oe 1 1. the electric flux through the cube, and Ans. $2 4 Se Gauss's Law in electrostatics states that the total electric flux through a closed surface enclosing a charge is equal to / / ey) times the magnitude of that charge. > b= f E.dS =4 5 £0 2. Net flux @ = Dy) + 9 where o= E. ds = 2a dS cos0® = 2aCx a?= 2070 @o= aC x a? cos]80° = —a7C @ = 2a3C +(-a3C) = aFC Nm? OW F 3. Net charge (q) = ey X ® = aCe coulomb q = a? Ceq coulomb Q. 11.Two uniformly large parallel thin plates having charge densities +o and -o are kept in the X-Z plane at a distance 'd' apart. Sketch an equipotential surface due to electric field between the plates. If a particle of mass m and charge ‘-g' remains stationary between the plates, what is the magnitude and direction of this field? — (CBSE Dethi 2011) Ans. wfft ttt tty Equipotential y surface | yg rd L.. ‘ ‘| f az -af/eeeeeee] The equipotential surface is at a distance d/2 from either plate in X-Z plane. For a particle of charge (—7) at rest between the plates, then 1. weight mg acts, vertically downward 2. electric force gE acts vertically upward. song = qk Bp ame q , vertically downward, i.e., along (-) Y-axis. Q. 12.Define intensity of electric field at a point. At what points is the electric dipole field intensity parallel to the line joining the charges? Ans. Intensity of Electric Field: The intensity of electric field at any point is defined on the electric force per unit charge when placed at that point te. > > E= lim -£ G9 qo The dipole field intensity is parallel to the line jaining at 1. axial points and 2. equatorial points. Q. 13.Two small identical electrical dipoles AB and CD, each of dipole moment ‘p' are kept at an angle of 120° as shown in the figure. What is the resultant dipole moment of this combination? If this svstem is subjected to electric field { -) directed along + X direction, what will be the magnitude and direction of the torque acting on this? — (CBSE Delhi 2011] Ans. Resultant dipole moment ‘pr = \p2 + p? + 2pp cos 120° =\2p? + 2p? cos 120° = (29? + 26)» (- ; ) =V2p2 -p=p making an angle 60° with Y-axis or 30° with X-axis. Y Torque, T= Bx E ( Tis perpendicular to both Band E) 1 =pEsin30°= E. p 2? Direction of torque is along negative Z-direction. Q. 14.Two concentric metallic spherical shells of radii R and 2R are given charges Q, and Q respectively. The surface charge Determine the ratio Q,:Q 2. (CBSE (F) 2013) Ans. Surface charge density o = constant Charge Q, = 41R2a Charge Q2 = 411(2R2)o ~% 4nReo 4 Q2 4n(2Ryro 4 Q. 15.4 spherical conducting shell of inner radius r, and outer radius 72 has a charge 'Q'. A charge 'g' is placed at the centre of the shell. = [CBSE (Al) 2010) 1. What is the surface charge density on the ee oipfarnn ii. outer surface of the shell? 2. Write expression for the electric field at a point x > rz from the centre of the shell. Ans. 1. Charge Q resides on outer surface of spherical conducting shell. Due to charge ¢ placed at centre, charge induced on inner surface is -q and on outer surface it is ty. So, total charge on inner surface —g and on outer surface it is Q + 4. Surface charge density on inner surface q dar Surface charge density on outer surface Tove FTED E(x) = Qty Q. 16, A small metal sphere carrying charge + Q is located at the eg es cd pee ntn) mebnee as shown in fig. Use Gauss's theorem to find electric field at points Py and Py. — [CBSE Delhi 2005] Ans. Now consider a spherical surface of radius rz , such that OP =r2 charge induced on inner surface of metallic sphere = - () ‘Charge induced on outer surface of metallic sphere = + Q .. Net charge enclosed by surface S» = 2 — Q = 0 .”. By Gauss Theorem Total electric flux through surface $2 = J / ¢9 * net charge enclosed. i E2. 4a? x0 2. Farr fp i.e., electric field at P» is zero. Q. 17.A thin straight infinitely long conducting wire having charge density A is enclosed by a cylindrical surface of radius r and length J, its axis coinciding with the length of the wire. Find the expression for the electric flux through the surface of the cylinder, = [CBSE (Al) 2077] Ans. Charge enclosed by the cylindrical surface = 4/ By Gauss Theorem, electric flux = / / e, x charge enclosed. =I / eg(Alp Q. 18.Find the electric potential due to a point charge. Ans. The electric potential due to a point charge: By definition electric potential at a point is the work done in bringing unit positive test charge from infinity to given point i.e., vn We ee) r ) 40 | at Phe 4 point at 4 dictance r fram point charae 7» Farce on 4 test charge (qq) at a distance x from point charge I ago ,F= he 4TEy x2 Work done in bringing test charge a small distance dx closer to point charge I 44 dW=F,dx cosl80° = — Fydx = — tM dx weg x2 “. Work done in nas point charge ¢g from infinity to a distance r | 440 1 r =- dx — 449 i: 4ney x? MEQ 46 4410 a 50 r wood *. Electric Potential , V = t qa tre 7 -q +9 +#—___ 2a ——_+ Find the electrostatic potential at equatorial point of an electric dipole. Ans. Let P be a point on the equator of an electric dipole formed of charges -q and +q at separation 2a. The distance of point P from centre of dipole =r ap =pp=VP 42 Electrostatic potential at P, Vp = Vp r- bam 2 yr That is electrostatic potential at each equatorial point of an electric dipole is zero. Q. 20.Deduce the expression for the potential energy of a system of Ans. two point charges g1 and q2 brought from infinity to the points vr and ¥ respectively in the presence of external electric field BE. Suppose ¢4 and ¢2 are two charges brought from infinity at locations Ty and 7) respectively in an external electric field. Let V( 71) and V( 7) be the potentials at positions 7 and 7 due to external electric field E . In this case work is done in bringing charges q4 and 42 against their own electric fields and external electric fields. Work done in bringing charge 4 from ° at location ¥) is W, = av ry) Work done in bringing gz against the electric field at location > is Wo =qoV( 7) Work done on 2 against the electric field due to q4 is 1 1 mig? Aneg S12 where rp2=1 7) - 77 .. Potential energy of system = Work done in assembling the configuration vit U =W, + Wo + Wa = VOR )+qaVirad +s ! 2.2 > Ira-nl Q. 21.Derive an expression for the potential energy of an electric dipole of dipole moment F in an electric field E Ans. The potential energy of an electric dipole of an electric field is defined as the work done in bringing the dipole from infinity to its present position in the electric freld. Suppose the dipole is brought from infinity and placed at orientation 4 with the direction of electric field. The work done in this process may be supposed to be done in two parts. 1 The work done (I’;) in bringing the dipole perpendicular to electric field from infinity. . Work done (1’z) in rotating the dipole such that it finally makes an angle 6 from the direction of electric field. . Let us suppose that the electric dipole is brought from infinity in the region of a uniform electric field such that its dipole moment D always remains perpendicular to electric field. The electric forces an charges +q and —q are qF and qf, along the field direction and opposite to field direction respectively. As charges +4 and - ¢ traverse equal distance under equal and opposite forces; therefore, net work done in bringing the dipole in the region of electric field perpendicular to field-direction will be zero, i.e. Wy =O +qe—e gE 24+ +—— From L infinity Now the dipole is rotated and brought ta orientation making an angle 6 with the field direction {i.e., 8, = 90° and 85 = 0°), therefore, work done W = pE{cos 0) — cos 05) = pE(cos 90° — cos 6) = — pB cas 0 -. Total work done in bringing the electric dipole from infinity, 7 e., Electric potential energy of electric dipole. U=W, +W2 = 0 — pE cos 8= — pE cos di) In vector form Q. 22.A test charge 'g' is moved without acceleration from A to C along the path from A to B and then from B to C in electric field E as shown in the figure. [CBSE (Al) 2012) 1, Calculate the potential difference between A and C. 2. At which point (of the two) is the electric potential more Ans. Ans. and why? . From the given diagram, potential difference between 4 and C Cc Ve-Va=-f EF. dheastso? = 8 x 4 = 48 A Hence, V; = V4 = 46 Vee Ky Because direction of electric field is in decreasing potential . An infinitely long positively charged straight wire has a linear charge density A em, An electron is revolving around the wire as its centre with a constant velocity ina circular plane perpendicular to the wire. Deduce the expression for its kinetic energy. . Plot a graph of the kinetic energy as a function of charge density A. [CBSE (F) 2073] . Infinitely long charged wire produces a radical electric field. A E= we (1) - 2Tegr The revolving electron exnerience an electrostatic force and provides necessarily centripetal force. 2 eE= ™™ (2) r eA mv2 2ieor r eh => m= 271g Kinetic energy of the electron eA 1 K=— my = 2 4TTég KE — Q. 24.An electric dipole is held in a uniform electric field. — (cBse (AN) 2012] 1. Show that the net force acting on it is zero. 2. The dipole is aligned parallel to the field. Find the work done in rotating it through the angle of 180° Ans. The dipole moment of dipole is | Pl =4 x (2a) Force on -g at A = —q E Force on +q at B Il + s be Net force on the dipole = ¢ E- q E=0 2. Work done on dipole W = AU = PE(cos 0) —cos 02) (cas? casi 80") =2PE I Q. 25.A point charge g is placed at O as shown in fig. Is Vp -Fq Ans. Ans. positive or negative when [CBSE (Al) 2006) 4. q>0 2. q <0. Justify your answer. o P Q 1. If q >0 (or positive), the potential at any point in electric field of gq will be positive and will go on decreasing as the distance of point from q increases, As OP < OQ, 50 Vp > Vo. “Vp = Va is positive. 2. If¢ <0 (or negative), the potential at any point in electric field of y will be negative and will increase as the distance of point from q increases; i.e., Vp < Vo orp — V’q is negative. 41. Plot a graph comparing the variation of potential '¥' and electric field ‘E' due to a point charge 'Q' as a function of distance 'R' from the point charge. [CBSE Delhi 2012] 2. Find the ratio of the potential differences that must be applied across the parallel and the series combination of two identical capacitors so that the energy stored, in the two cases, becomes the same. 1. The graph of variation of potential and electric field due to a point charge Q with distance R from the point charge is shown in fig. Oo Les Le In series arrangement net capacitance C, = C / 2. In parallel arrangement, net capacitance, C p= 2C Energy stored LU = 1 / 2 CV? If Vg and V, are potential differences applied across series and parallel arrangements, then given Uy =U, Uy 1.2 1.42 > SCV? =/. 4. The charge Q =CI’, V = same, C = increases; there, charge on plates increases. 5. Energy stored by capacitor U' = I / 2C'V?, also increases. Q. 30.Two identical parallel plate (air) capacitors C; and Cz have capacitances C each. The space between their plates is now filled with dielectrics as shown. If the two capacitors still have equal capacitance, obtain the relation between dielectric constants K.K; and K>. (CBSE IF) 2011) 2 2 d Ans. Let A — area of each plate. Let initially E_yA Ccy=C= ' d =Cs After inserting respective dielectric slabs: Cy) = KC fi) and Eyl f Ko Sy(A / . Co = ata . Ko =u Eyl (Ky + Ki 7 2} ~ 2d K2) a a Clo = OK + Ko) (ii) Mg = 5 (Ks 2 (ii) From (i) and (ii) Cy =C's - Co. . KC = 2 (Ky + K2) i K = 5 (K) + Ko) Q. 31.You are given an air filled parallel plate capacitor C; . The space between its plates is now filled with slabs of dielectric constants K; and Ky as shown in Cz . Find the capacitances of the capacitor C» if area of the plates is A distance between the platesisd. (CBSE (F) 2011] K2 =e ~~ d d 1 1 | d d Q. 32.You are given an air filled parallel plate capacitor C; . The space between its plates is now filled with slabs of dielectric constants K; and Ky as shown in C2 . Find the capacitances of the capacitor C2 if area of the plates is A and distance between the plates isd. — [CBSE (F) 2011) Cy Cc. Ans. C= eg d Cy = Reet) Reent / d d Ay +k (= €; (Ky al * 2 Other Important Questions Q. 33.Define electric field strength. P2) &ye e[K, + Kaj 2d Is it a vector or a scalar quantity? Ans. The electric field strength at a point in an electric field is defined as the electrostatic force on a unit positive charge when placed at that paint and its direction is along the direction of electrostatic force. Electric field strength is a vector quantity. Q. 34.Two charges g and —3q are placed fixed on x-axis separated by distance 'd'. Where should a third charge 2q be placed such that it will not experience any force? (NCERT Exemplar) Ans. At P: On 27, force due to q is to the left and that due to —3y is to the q-———_ « ———> eee -3q right. Pp aq yy 2g 6q? fregx? — Grreg(dd +x)? (d +x}? = 3x? 2x? —2dx — d? = 0 > vag V5 2 2 (-ve sign would be between q and -3q and hence is pe 5 1 + Vid) to the left of q > yet, Vu 22 Q. 35.An electric dipole is free to move in a uniform electric field. Explain its motion when it is placed 1. parallel to electric field 2, perpendicular to electric field. Ans. Net force on dipole in uniform electric field = 0 Net tarque on dipole in uniform electric field tr = pF sin@ 1. When dipole is placed parallel to electric field, @ = 9, sin @ = 0, so torque z = @. Thus when an electric dipole is placed parallel to electric field, net force and net torque on dipole is Zero; so dipole remains al rest. . When dipole is placed perpendicular to electric field, 0 = 90", so torque cr = pF sin 90° = pL. This torque tends to align the dipole parallel to the direction of electric field and due to inertia, the dipole will start executing oscillatory motion Q. 36.A very thin plate of metal is placed exactly in the middle of the two plates of a parallel plate capacitor. What will be the effect ‘on the capacitance of the system? Ans. Fora metal K = ~ and so when? < < d_, the capacitance £9A fA &q c= d-s| I tA Ast< 0 and E= —Ei,N / C for x <0 are given. A right circular cylinder of length Acm and radius r cm has its centre at the origin and its axis along the X-axis. Find out the net outward flux. Using Gauss's. law, write the expression for the net charge within the cylinder. [CBSE Delhi 2008(C}] Ans. Electric flux through flat surface 31 a = le Ey dS, g (Fxt)- (dSyi) = ES, Electric flux through Mlat surface Sy CB, i)- CdS) = =f, Blectrie flux through curved surface Sy (By d8,)=[, Fy dS, cos 90° 1. Net electric flux, 6 =) + = Bx (5, +) But (rx 108? me = ne? x 10-4 ma? @= Ey .2Gr? x10) By Gauss’s law => q=ey0= egB, Qn x104) ‘0 (E,P «104 Ce] gE er” x10 =4neg ot 56E,7? 10" coulomb. 2 Q. 10.The graph shows the variation of voltage 'l" across the plates of two capacitors A and B versus increase of charge 'Q' stored on them. Which of the two capacitors has higher capacitance? Give reason for your answer. — (CBSE Delhi 2004) Ans. OQ” I I Slope of line As slope of 4 is smaller, capacitance of 4 is higher. Q. 11.The given graph shows the variation of charge g versus Ans. potential difference V for capacitors C; and Cz. The two capacitors have the same plate separation, but the plate area of C2 is double that of C7. Which of the lines in the graph correspond to C; and C2 and why? = [CBSE Delhi 2006) HA d ‘Capacitance of parallel plate capacitor C = As plate area of C> is double that of C;; C2 =2C, Slope of g —l’ graph = g/V =C As slope of 4 is greater than slope of B, A is corresponding to larger capacitance and B to smaller capacitance. So lines corresponding to C; and Cz are B and A respectively. Q. 12.Two charges 5 nC and -2 nC are placed at points (5 cm, 0, 0) and (23 cm, 0, 0) in a region of space where there is no other external field. Calculate the electrostatic potential energy of this charge system. = (CBSE Delhi 2008C) Ans. Given gq) = SnC = 5 «x 107° C,qgo9 = -2nC = -2 K10-%C The charges are placed on X-axis. The distance between the charges X= Ng — Np = (23 — F)om = 18 em = O18 mw .. Electrostatic potential energy of charges yt ae 4dr x 9x 109 (5 x 10-8)(-2 x 1n-? d= -7 =5 x10 O18 J Q. 13.A slab of material of dielectric constant K has the same area as that of the plates of a parallel plate capacitor but has the thickness d/2, where d is the separation between the plates. Find out the expression for its capacitance when the slab is inserted between the plates of the capacitor. [CBSE (Al) 2013] Ans. Capacitance with dielectric of thickness 't’ Area =A fA t Separation '‘¢ rpenealtt par Vacuum LONG Answer Questions 5 Marks Q. 1. Find expressions for the force and torque on an electric dipole kept in a uniform electric field. (cBsE (Al) 2008) OR An electric dipole is held in an uniform electric field 1. using suitable diagram show that it does not undergo any translatory motion, and 2. derive an expression for torque acting on it and specify its direction. [CBSE Delhi 2005] Ans. Consider an electric dipole placed in a uniform electric field of strength E in such a way that its dipole moment B makes an angle with the direction of E. The charges of dipole are — q and + g at separation 2! the dipole moment of electric dipole, p=qal Force: The force on charge + ¢ is, Fi =4 BE, along the direction of field E The force on charge - g is Poe gE, opposite to the direction of field E. Obviously forces Fi Foand are equal in magnitude but opposite in direction; hence net force on electric dipole in uniform electric field is As net force on electric dipole is zero, so dipole does not undergo any translatory motion. Torque: The forces Fiand Foform a couple (or torque) which tends to rotate and align the dipole along the direction of electric field. This couple is called the torque and is denoted by T. -. Torque ~ Magnitude of one force * perpendicular Tt distance between lines of action of forces =qE (BN) =qE (21 sin 8) =(q 21) E sin @ =pE sin [using ()] (it) Clearly, the magnitude of torque depends on orientation (#) of the electric dipole relative to electric field. Torque (z) is a vector quantity whose direction is perpendicular toboth pand E. In vector form e ‘px E - (itt) Thus, if an electric dipole is placed in an electric field in oblique orientation, it experiences no force but experiences a torque. The torque tends to align the dipole moment along the direction of electric field. Maximum Torque: For maximum torque sin @ should be the maximum. As the maximum value of sin 0 =/ when @ = 90° .. Maximum torque. tray = pF Q.2. An electric dipole of dipole moment p is placed in a uniform electric field E. Write the expression for the torque a experienced by the dipole. Identify two pairs of perpendicular vectors in the expression. Show diagrammatically the orientation of the dipole in the field for which the torque is 41, Maximum 2. Half the maximum value 3. Zero. Ans. —____-_» +q + E p -q —— (i) Maximum torque (ili) Torque experienced by an electric dipole Q.3. Ans. Te px E Pairs of perpendicular vectors (i) ( tT, PY (ii) ( Tr E) 1. Magnitude of torque r= pF sin @ For maximum torque (sin Og, =! > @= 90° 2. For T= 5 Tuan pE sin = Loe I > Sine O=5 or 8 = 30° 3. For zero torque, sin€ =0>6=0 The orientations are shown in the figure. Find an expression for the work done in rotating an electric dipole in a uniform electric field. Let an electric dipole be rotated in electric field from angle 6; to 8» in the direction of electric field. In this process the angle of orientation 8 is changing continuously; hence the torque also changes continuously. Let at any time, the angle between dipole moment ‘pand electric field FE be a, then torque on dipole r= pe sina The work done in rotating the dipole a further small angle da is diV=Torque * angular displacement Spek sin ada Total work done in rotating the dipole from angle 8, to 85 will be the Ans. sum of all such small works which is found by integration “ 85 x Oy W=("? pE = pE|-c 2 fh, pE sin 01 da. = pE[-cosa],” = — pE [cos 02 — cos 6) | = pE (cos 8; — cos 83) wi) Special case : If electric dipole is initially in a stable equilibrium position (8, = 0°) and rotated through an angle 6(82 = 8), then work done W = pE (cos & — cas 0) = pE (1 — cas 0) (ii) . Find an expression for the electric field strength at a distant point situated 1. on the axis and 2. along the equatorial line of an electric dipole. = [case (F) 2009) Consider an electric dipole 4B. The charges - q and + gq of dipole are situated at 4 and B respectively. The separation between the charges is 2, Electric dipole moment, fr = ¢.2/ The direction of dipole moment is fram — q to + gq. (i) At axial or end-on position: Consider a point ? on the axis of dipole at a distance r from mid point O of electric dipole The distance of point P from charge +, is BP=r-1 and distance of point P from charge - is, AP =r + /. Let £; and £2 be the electric field strengths at point P due to charges + q and — ¢ respectively. We know that the direction of electric field due to a point charge is away from positive charge and towards the negative charge. Therefore E ! 1 from B to P. cy = rem B to dren (r—l)? ; and I q E9= (from P to A 2° deg (r+)? (fre } Clearly the directions of electric field strengths Ey E> and are along the same line but opposite to each other and E; > Es because positive charge is nearer. “. The resultant electric field due to electric dipole has magnitude equal to the difference of Ey, and Eg and direction from B to P i.e. 1 q I qg Fig, (r— 1)? Ate (vr + LP 4 i t AT éy [ -W t wl 4 (r+lP —(r-1P? 7 ATE y [ 408, / E=B;- Fo = _ 4 rl 1 2(q.2br tre, (P= Pye Amey (= PP But ¢.2/ = p (electric dipole moment) I pr B= (i) deg (y2—[2)2 If the dipole is infinitely small and point /? is far away from the dipole, theny >> /, therefore equation (7) may be expressed as 1 pr 1 2p = or EF 3 4diey yt die 73 (ii) This is the expression for the electric field strength at axial position due to a short electric dipole. (ii) At a point of equatorial plane :Consider a point P on broad side ‘on the position of dipole formed of charges +q and - q at separation 2i. The distance of point P from mid point (OQ) of electric dipole is r. Let £; and £2 be the electric field strengths due to charges +, and -q of electric dipole. From fig. AP = BP =V r2 +12 E=E,cos0+E,cos0 E« i 4 E> 4Ameg y? + [7 1 q °2 0 firey 2 4? along B to P along Plo A Clearly -, and E,are equal in magnitude ?.¢ | Edl=| Exlorey = £2 To find the resultant of Ey and Ee ; we resolve them along and perpendicular to AB. ‘Component of Eyalong AB = E, cas @, parallel to B a ‘Component of E; perpendicular to AB = E, sin @ along 0 to P ‘Component of Egalong AB = E2 cos 6, parallel to B A ‘Component of Es perpendicular to AB = Fs sin Galong P to 0 Clearly components of Ei and E> perpendicular to AB: £; sin@ and EF sin 6 being equal and opposite cancel each other, while the components of E, Eyand along 4B: E ; cos dandF > cos 8, being in the same direction add up and give the resultant electric field whose direction is parallel to B A Resultant electric field at P is E = £; cos8+ £2 cos® 1 q But = day (+2) OB f I and cos = = = PROV +2 (2 + PHP q l E = 2E; cos 0= 2 x frre, (2 + 2) pr? 4 2p? I 2Qgl Athy 2 4 232 But 9.2! = p = electric dipole moment I 241 trey (2+ BP? If dipole is infinitesimal and point P is far away, we have/ < <7, so 1? may be neglected as compared to r? and so equation (3) gives 1 1 P (iti) E drag (y2)3!2 4weg 73 i.e. electric field strength due to a short dipole at broadside on Ans. i P . parallel lo B a (tv) 3 trey a Its direction is parallel to the axis of dipole from positive to negative charge. Itis clear from equations (ii) and (iv) that electric field strength due to a short dipole at any point is inversely proportional to the cube of its distance from the dipole and the electric field strength at axial position is twice that at broad—side on position for the same distance. . Derive the expression for the electric potential at a point due to an electric dipole. Mention the contrasting features of electric potential of a dipole at a point as compared to that due toa single charge. = (CBSE Delhi 2008, 2007) Consider an electric dipole having charges - ¢ and + ¢ at separation ‘2a’, The dipole moment of dipole isp =q(2 a ),directed from -g to +q. The electric potential due to dipole is the algebraic sum of potentials due to charges +q and -4 lfr; and rg are distances of point P from charge + ¢ and — grespectively, then the potential due to electric dipole at point P, A AY Fg —p 2s yo i 8 Log. q ot dre 1p Are po ims by r9 / ..(i) If (r, 8) are polar coordinates of point ? with respect to mid-point O of dipole, then By geometry, ry? =r? +a? - Qarcos 0 ve (ii) and, ry? = v2 + a? + ar cos 0 2 2a cosB | a? From (i), ry Pfr + / r re a ae - - Ifr>>ai.e., — <<1, then it is sufficient to retain terms only upto first r order in ( a ). . 2 2 1 2a cos °] =r "I 2 => Aime [ _ 2a cos °] can) r Similarly, rar? ! c. 2acose] r v2 2 = ry ri 2eene| _v) r From (iv) and (¥), 11 ! _ 2a cos y" ry r r -U2 ais 11), , 2a cos MM r r . . sa . a Using binomial theorem and retaining terms upto first order in r ‘only, we have + att _(-1)2ee88) 114 Zo0] (vi) ie sls 2 r ie r and, tath or) _Avii) " r = Substituting these values in (i), we get (1+4cose TF 4dreg = 41s Sc080-1 dney rl or 1 (q.2a) cos dnp 2 I cos. or, v pes se(viti) 4reg or = But, peost=p.P where, # is unit vector along position vector OP Electric potential due to an electric dipale is ce 122 a oF V=— > (forr >> a) =——E 4neg r? 4neg 73 Constrasting features: The electric potential due to a dipole depends on distance r and also on the angle between position vector and dipole moment Pp. The electrostatic potential at large distances falls off, as 1 /r? and not as 1/ r which is the characteristic due to a single charge. Special Cases: 1. When point P lies on the axis of dipole, then8=0°.”. cas @=cos @ =I 1 p a ves p dren 2 2. When point P lies on the equatorial plane of the dipole, then 6=90" “. cos O=cos 90° = 0 ved DT EB Géntn and Denve farce thanram in slarétenctatine Ans. Statement of Gauss's Theorem: The net—oudward normal electric flux through any closed surface of any shape is equal to 1/ eq times the total charge contained within that surface, t.e., => Direction 5S 4 of normal ae a §, Eeds =—%q . £0 where fs indicates the surface integral over the whole of the closed surface, Sq is the algebraic sum of all the charges (?.¢., net charge in coulombs) enclosed by surface S Proof of Gauss's Theorem: Let a point charge + 7 coulomb be placed at O within the closed surface. Let # be the electric field strength at P. Let OP =r and the permitivity of free space or vacuum be ey Consider a small aread ‘S of the surface surrounding the point? . Then the electric flux through ¢ ‘Sis given by tn Be @ But the electric field strength at q P, I Gin i 1 (Sinee wnit vector * = FE dey 12” — Frey’ 7 Tir) I Fed S_ 4 d= Be I> Gra, 4 SO rey do where ye dS cos 0 dQ T é S_ m r is the solid angle subtended by area dS at point O. Here @ is the angle between d ‘Sand E. Hence electric flux through whole of the closed surface - 2 > q " o= > _E dS = x pdO 7s 4néq But faa is the solid angle due to the entire closed surface § at an internal point O = 4a .o- 4 4 i Oday" ag! If there are several charges, + ¢,, + q2. + 43-0. —- 97. — q2'- — q;'...inside the closed surface, each will contribute to the total electric flux. For positive charges the flux will be outward and hence positive; for negative charges the flux will be inward and negative. Therefore, the total electric flux in such a case 1 I I a as Sy he 9g F999 0 £0 =o £0 eo £0 £0 ! toa? : =— (4 +42 + 93 4-4 42! 493" -- eo where Sy is the algebraic sum of the charges within the closed surface. Hence total eleciric flux through any closed surface is equal to 1/ &q times the total charge (in coulomb) enclosed within the surface which is Gauss's law. If the charge is uniformly distributed, then Gauss's theorem takes the form §, Bad =, 3 J, e-av & °¥ Py where p is the charge density, dV’ is a small element of the volume and the volume integral is taken through the volume bounded by surface S. Itis interesting to note that Gauss's theorem remains valid as such even for charges in motion. Moreover, it is applicable to any field obeying inverse square law. 1. Using Gauss Theorem show mathematically that for any point outside the shell, the field due to a uniformly charged spherical shell is same as the entire charged shell is concentrated at the centre. 2. Why do you expect the electric field inside the shell to be zero according to this theorem. = [C&sE (Al) 2007] OR A thin conducting spherical shell of radius R has charge Q spread uniformly over its surface. Using Gauss's theorem, derive an expression for the electric field at a point outside the shell. (CBSE Delhi 2009) Draw a graph of electric field Z(r) with distance r from the Ans. centre of the shell for? = r = &. OR 1, Using Gauss's theorem derive the expression for the electric field intensity at a point outside a uniformly charged thin spherical shell of radius R and surface charge density o C/m 9. — (CASE Delhi 2008, 2004, (Al) 2007] 2. Use Gauss's theorem to find the value of electric field intensity at a point inside this shell (or hollow charged conducting sphere)? OR Using Gauss's law obtain the expression for the electric field due to a uniformly charged thin spherical shell of radius R at a point outside the shell. Draw a graph showing the variation of electric field with r, forr > Randr <.R. (CBSE Delhi 2014, (Al) 2013] 1. Electric field intensity at a point outside a uniformly charged thin spherical shell: Consider a uniformly charged thin spherical shell of radius R carrying charge (2. To find the electric field outside the shell, we consider a spherical Gaussian surface of radius r(>R), concentric with given shell. If E is electric field outside the chall than hu eummatry alactric field ctrannth hac cama magnitude Eg on the Gaussian surface and is directed radially outward. Also the directions of normal at each point is radially outward, so angle between E jand d ‘Sis zero at each point. Hence, electric flux through Gaussian surface as > p, =Eoed. $=Ep dS cosO=Ey .4nr? Now, Gaussian surface is outside the given charged shell, so charge enclosed by Gaussian surface is Q. Hence, by Gauss's theorem > + 4 f, EgedS x charged enclosed ) Q x 1 > Eo 4ar~ =—x Q=> Ey= £0 4neg r° Thus, electric field outside a charged thin spherical shell is the same as if the whole charge Q is concentrated at the centre. If o is the surface charge density of the spherical shell, then Q =4rR?o€ pee! daha — Ra “8 trey 2 Electric field inside the shell (hollow charged conducting sphere): The charge resides on the surface of a conductor. Thus a hollow charged conductor is equivalent to a charged spherical shell. To find the electric field inside the shell, we consider a spherical Gaussian surface of radius r (< R), a oe a the shell, then by symmetry electric field strength has the same magnitude £; on the Gaussian surface and is directed radially outward. Also the directions of normal at each point is radially outward, so angle between -E jandd “Sis zero at each point. Hence, electric flux through Gaussian surface =[ Ej dS cosO=5; .4nr? Now, Gaussian surface is inside the given charged shell, so charge enclosed by Gaussian surface is zero. Hence, by Gauss's theorem 2? 2 |B => E; 4nr? Thus, electric field at each point inside a charged thin spherical shell is zero. The graph is shown in fig. Q. 8. State Gauss the o rem in electrostatics. Ap ply this the o rem to ob tain the ex pres sion for the elec tric field at a point due to an in finitely long, thin, uniformly charged straight wire of lin ear charge den sity A Cm—!, — (cBse Deini 2009; (Al) 2008, 2007, 2005) Ans. Gauss Theorem: Refer to point 11 of Basic Concepts. Electric field due to infinitely long, thin and uniformly charged straight wire: ‘Consider an infinitely long line charge having linear charge density 4 coulomb metre ~* (linear charge density means charge per unit length). To find the electric field strength at a distance r, we consider a cylindrical Gaussian surface of radius r and length / coaxial with line charge. The cylindrical Gaussian surface may be divided into three parts : 1. Curved surface $; 2. Flat surface $5 and 3. Flat surface $5. By symmetry the electric field has the same magnitude E at each point of curved surface §; and is directed radially outward. We consider small elements of surfaces S$) 52, and 53. The surface element vector d $y is directed along the direction of electric field (ie .., angle between Eandd “S , is zero); the elements d gs, andd 83 are directed perpendicular to field vector Eli e.., angle between d Ss and Eis 90° and so also angle betweend $3 and E) Electric Flux through the cylindrical surface d824f OF 6 [BS cons J, £05, +040 Sef a) pimeuccwensd mine mec wenpanot carved mnie =E2nel (since arenof curved surface = 2n rl) As 4 is charge per unit length and length of cylinder is /;, therefore, charge enclosed by assumed surface =(Al} By Gauss’s theorem -2 > 4 p E ed § =—-» charge enclosed - & 1 a => E.2nrl=—(A) => E= £q 2negr Thus, the electric field strength due to a line charge is inversely proportional to r. Q.9. 1. Define electric flux. Write its S.1. units. 2. Using Gauss's law, prove that the electric field at a point due to a uniformly charged infinite plane sheet is independent of the distance from it. 3. How is the field directed if i. the sheet is positively charged, ii. negatively charged? (CBSE Delhi 2012, 2007, 2005, 2004] Ans. 1. Electric flux: It is defined as the total number of electric field lines passing through an area normal to them Also,o=$ Bad 3 The SI unit is Nm?/C or volt-metre. Let electric charge be uniformly distributed over the surface of a thin, non-conducting infinite sheet. Let the surface charge density (/.e., charge per unit surface area) be o. We have to calculate the electric field strength at any point distance r from the sheet of charge. To calculate the electric field strength near the sheet, we now consider a cylindrical Gaussian surface bounded by two plane faces 4 and B lying on the apposite sides and parallel to the charged sheet and the cylindrical surface perpendicular to the sheet (fig). By symmetry the electric field strength at every point on the flat surface is the same and its direction is normal outwards at the points on the two plane surfaces and parallel to the curved surface. Total electric flux fF -08) = §g as, + fe as, + fp eas, or fe as Jj, Ease cos0® +f, EadSz cos0? + [. BdSs cos 90° HE o5 +8 dS, =Ea4Ean2Ea -. Total electric flux = 26 a. As ais charge per unit area of sheet and a is the intersecting area, the charge enclosed by Gaussian surface = oa According to Gauss's theorem, I Total electric flux = — (total charge enclosed by the surface) ay I Le, 2Ea= (aa) £9 . 6 E= 2eq (oct) Thus electric field strength due to an infinite flat sheet of charge is independent of the distance of the point and is directed normally away from the charge. \f the surface charge density o is negative the electric field is directed towards the surface charge. 3. i. Away from the charged sheet. ii. Towards the plane sheet. Q. 10.Apply Gauss's Theorem to find the electric field near a charged ‘conductor. OR Show that the electric field at the surface of a charged oA . 5 conductoris F= n where ois surface charge density and #0 2% is a.unit vector normal to the surface in the outward direction. (CBSE (Al) 2010) Ans. Let a charge Q be given to a conductor, this charge under electrostatic equilibrium will redistribute and the electric field inside the conductor is zero (i. ¢., £,,, = (@). Let us consider a point P at which electric field strength is to be calculated, just outside the surface of the conductor. Let the surface charge density on the surface of the conductor in the neighbourhood of P be s coulomb/ metre?. Now consider a small cylindrical box CD having one base ( passing through /? ; the other base P lying inside the conductor and the curved surface being perpendicular to the surface of the conductor. Let the area of each flat base be a. As the surface of the conductor is equipotential surface, the electric field strength F at P, just outside the surface of the conductor is perpendicular to the surface of the conductor in the neighbourhood of P. The flux of electric field through the curved surface of the box is zero, since there is no component of electric field £ normal to curved surface. Also the flux of electric field through the base D is zero, as electric field strength inside the conductor is zero. Therefore the resultant flux of electric field through the entire surface of the box is same as the flux through the face C. This may be analytically seen as: IFS; and S» are flat surfaces at C and D and $3 is curved surface, then Total electric flux § S =f, 2 dS1 =f, Ea a Od J, #a8's cos 90" =f, E dS, =Ea As the charge enclosed by the cylinder is (aa) coulomb, we have, using Gauss's theorem, 1 Total electric flux = x charge enclosed _ I faa) ~ 7 gy or “ &y (i) Thus the electric field strength at any point close to the surface of a charged conductor of any shape is equal to i/ &g times the surface charge density o. This is known as Coulomb's law. The electric field strength is directed radially away from the conductor if o is positive and towards the conductor if cis negative. lf is unit vector normal to surface in outward direction, then a it £0 Obviously electric field strength near a plane conductor is twice of the electric field strength near a non—conducting thin sheet of charge. Q. 11.Briefly explain the principle of a capacitor. Derive an expression for the capacitance of a parallel plate capacitor, whose plates are separated by a dielectric medium. — [CBSE Delhi 2004C, 2002] Ans. Principle of a Capacitor: A capacitor works on the principle that the conductor is brought near it. Thus a capacitor has two plates separated by a distance having equal and opposite charges. Parallel Plate Capacitor: Consider a parallel plate capacitor having two plane metallic plates 4 and B, placed parallel to each other (fig.) The plates carry equal and opposite charges + Q and - Q respectively. In general, the electric field between the plates due to charges + Q and - Q remains uniform, but at the edges, the electric field lines deviate outward. If the separation between the plates is much smaller than the size of plates, the electric field strength between the plates may be assumed uniform. +Q -Q tet eee eeegees Suppose 4 be the area of each plate, 'd' the separation between the plates, K the dielectric constant of medium between the plates. If cis the magnitude of charge density of plates, then of The electric field strength between the plates o b= where ep=permittivity of free Ke Space. wu(i) The potential difference between the plates, Van = Ed = ol fli) Vag = Ed Ken Putting the value of co, we get (Q/Ayl Qa Keg K gpd Capacitance of capacitor, Q Q © Vin (QdiKepA) or C= ne .. (tit) d Var= This is a general expression for capacitance of parallel plate capacitor. Obviously, the capacitance is directly proportional to the dielectric constant of medium between the plates. eo A For air capacitor (K =1); capacitance C = . This is expression d for the capacitance of a parallel plate air capacitor. Obviously, the capacitance of parallel plate (air) capacitor is : 1, directly proportional to the area of each plate. 2. inversely proportional to the distance between the plates. 3. independent of metal plates. Q. 12.Find an expression for the capacitance of a parallel plate capacitor when,a dielectric slab of dielectric constant K and thickness ¢ = 2 but of same area on the plates is inserted Ans. between the capacitor plate. (d = separation between the plates). [CBSE (F) 2010) Consider a parallel plate capacitor, area of each plate being 4, the separation between the plates being 7. Let a dielectric slab of dielectric constant K and thickness / < d be placed between the plates. The thickness of air between the plates is (d — 1). If charges ‘on plates are + Q and - @, then surface charge density +Q -Q Alj+ 7 Ui} —/jB + K es + = + - + + = + ih - + a 4 tl : d } -2 | oc The electric field between the plates in air, Eyp= = a Q ey The electric field between the plates in slab, Eo = = & Q KepA we lb) The potential difference between the plates V4 = work done in carrying unit positive charge fram one plate ta another = Ex (as field between the plates is not constant). =E, (d-t)+ Ey =—2-(a-1) 42-4 BoA KeoA Q 1 Vag =——| d-t+— AB spd K ‘Capacitance of capacitor, C = ae = — 2 Yap 2 ( d-ra } EA K or, g-Sk___ d-t+— a-i{ 1-2] K Here, t =< egA &oA Q. 13.Find expression for equivalent capacitance of three capacitors when connected 1. in series and 2. in parallel. Ans. In fig. (c) three capacitors of capacitances C1), Co, C3 are connected in series between points 4 and D. +0 -Q +0 -Q +Q 2 3 2 3 ae A ze eS 4 ||+ c,-|- 2 —_l+ o, -[|- 2 —+ o, -||- 0 : pe fae ert kidok: VN “Vv, Vo +e e+ + ° 1 sl ame. —_—__ y > (b) For first capacitor, Vi=Va-Vep=Q wali) Cy For second capacitor, V2=Ve-Vo=Q wil) Cy For third capacitor, V3=Ve-Vp=Q Aili) C3 Adding equations (i), (iz) and (r/7), we get 1 1 1 V, + V5 +V3 =Vy -Vp =| —t+— + — wei pet Vga hp -| GG, Cs lf be the potential difference between A and D, then Vi-Vp=v -. From (4), we get v=, +¥, +v)-9[ 14141] Ai If in place of all the three capacitors, only one capacitor is placed between 4 and 2) such that on giving it charge (2, the potential difference between its plates become I’, then it will be called equivalent capaciter. If its capacitance is C, then v2 C -.(vi) Comparing (v) and (wi), we get ~lvii) Thus in series arrangement, "The reciprocal of equivalent capacitance is equal to the sum of the reciprocals of the individual capacitors." Parallel Arrangement: In fig. (c) three capacitors of capacitance ( , , C2, C3 are connected in parallel. 42 5 -@ at. le - Lid & Vv @) (a In parallel the potential difference across each capacitor is same I” (say). Clearly the potential difference between plates of each capacitor V4 - Vg = F (say) The charge Q given to capacitors is divided on capacitors C; . C2, C3. Letg,; . 92. q3 be the charges on capacitors C ; respectively. Then Q=q1 + 92443 = (i) and qr =C1V, gz =CoV, q3 =C3V Substituting these values in (i), we get Q=C)V + Cob + C5V or QO =(Cy + Co + Cg) ¥ .. {li} If, in place of all the three capacitors, only one capacitor of capacitance C be connected between A and #; such that on giving it charge Q, the potential difference between its plates be I’, then it will be called equivalent capacitor. If C be the capacitance of equivalent capacitor, then OQ =CV (iii) ‘Comparing equations (ii) and (#7), we get CV =(Cy + Co +03) V Cy + Ca+ C3 --(tu} or Cc Q. 14.Derive an expression for the energy stored in a parallel plate Ans. capacitor C, charged to a potential difference V. Hence derive an expression for the energy density of acapacitor. = [C8SE (A!) 2012, 2008, 2008C] When a capacitor is charged by a battery, work is done by the charging battery at the expense of its chemical energy. This work is stored in the capacitor in the form of electrostatic potential energy. Consider a capacitor of capacitance C. Initial charge on capacitor is zero. Initial potential difference between capacitor plates = zero. Let a charge ( be given to it in small steps. When charge is given to capacitor, the potential difference between its plates increases. Let at any instant when charge on capacitor be ¢, the potential difference between its plates ’ = : e+e eee + tl SV 45 = Now work done in giving an additional infinitesimal charge dq to capacitor dW =V dg = i dq The total work done in giving charge from 0 to ( will be equal to the sum of all such infinitesimal works, which may be obtained by integration. Therefore total work De, i a; Mo ee If V is the final potential difference between capacitor plates, then Q = CV ye I w= fF 7 oye! av 20 2 2 This work is stored as electrostatic potential energy of capacitor i.e., 2 i i Electrostatic potential energy, Us Qe Crs av 20 2 2 Energy density: Consider a parallel plate capacitor consisting of plates, each of area A, separated by a distance d. If space between the plates is filled with a medium of dielectric constant K, then Capacitance of capacitor, _ Keg d If sis the surface charge density of plates, then electric field strength between the plates o E= > cK ek Key Charge on each plate of capacitor @ = oA = K eg EA ge _ (Keak .”. Energy stored by capacitor, Uz = 20 2KepAfd) = 2 =Kepk?Ad But 4d = 7, volume of space between capacitor plates . I. 22 .. Energy stored, Us > Kegk-Ad U I Electrostatic Energy stored per unit volume, wv, = => Kejk? This is expression for electrostatic energy density in medium of dielectric constant Kk. . . ! In air or free space (K =1), therefore energy density, «1, = > Eg? Q. 15.Explain the underlying principle of working of a parallel plate capacitor. If two similar plates, each of area A having surface charge densities + o and - o are separated by a distance 'd' in air, write expressions for 1. the electric field at points between the two plates. 2. the potential difference between the plates. 3. the capacitance of the capacitor so formed. Justify your answer in each case, = [CBSE (Al) 2007) Ans. A B +e ee e+ Principle of capacitor: A capacitor works on the principle that the capacitance of a conductor increases appreciably when an earthed conductor is brought near it. A parallel plate capacitor consists of two metallic plates 4 and B separated by a distance 1". Action of the earthed plates B. Let plate 4 be given a positive charge q. When an earthed plate 2 is brought near it, a negative charge (- ¢) is induced on the inner side and a positive charge (+ 4) is induced on the outer side of plate B: due to being earthed the charge + a induced as # is transferred to earth, so due to presence of (- 4) charge on plate 5, the potential of plate 4 is appreciably reduced, so capacitance of conductor C = 4 is appreciably increased; thus a I parallel capacitor has two metallic plates having charges + 4 and - ¢. o 1. Electric field between two plates, E = & . al 2. Potential difference between the plates, |’ = Ed = £9 aA 3. Capacitance of capacitor C = d Q. 16.Two capacitors with capacitances C, and Cy are charged to potentials V7 and V2 respectively and then connected in parallel. Calculate the common potential across the combination, the charge on each capacitor, the electrostatic energy stored in the system and the change in electrostatic energy from its initial value. Ans. Q, Cc. ¥, + Q, C3, V2 Suppose two charged capacitors C; and C2 charged to potentials V) and ’2 are connected in parallel, with their positive terminals connected together and negative terminals connected together as shown in the figure. After connection, the charge redistributes in such a way that the potential differences across C; and C2 become equal. Charges on capacitors before connection, Q; =C;V;, Qo =CoV3 J, + Q. ‘Common potential after connection, ’ = Qu + Qo C, + C> VyCy + Colo vel a (i) Cy + Cg If Q; and (J> are charges after sharing, then Q,'=C,V, Q,'=C2V > vec Qr' C2 This means that after connection, the charges on capacitors are Shared in ratio of their capacitances. di) Electrostatic energy stored in the system : When charges are shared between two capacitors, then some energy is dissipated as heat and hence, definitely there is a loss of energy. The energy loss = Initial energy (U’; ) — Final Energy (U’;) Initial energy: After connecting the two capacitors, their combined capacitance is (C; + C2 ) and common potential is I’; therefore, final electrostatic energy u; $e +C,)V? {using (7) C, +Cp (CY, + Ca¥)? at 2 G+ <. Loss in energy during sharing of charges, 1 2 SCV, +02V3)" g(CiM +C2¥a) eve +e? 2 poe C402 (Cy HCaM(CWV, ? + CpVy?)—(CV, + C2V 2)? 2, +Cy) €,C2¥,' + CC; C12 2, +c) | ie, Energy loss, A\ As ©; Co, and (V’; ~ ’2) are all positive, therefore, AU’ is always positive. Thus, in the process of redistribution of charges, there is always a loss of energy. This energy is lost in the form of heat in connecting wire. IMPORTANT FORMULAE 1. Coulombs force I 4142 dvegK 2 For air between the charges, dielectric constant K =1. T4192 dope, 9 Fair = 2. = Electric field strength E = lim = 9079 Go 1. Electric field strength due to a point charge “4h 1 # 1 = ‘* a) sg BLN Oy 0 ass whore fis unit vector long 73; f= 4g rr? Ame 3 2. Electric field strength due to a charged conducting sphere ora spherical shell of radius R i. inside the shell, r < , Ej, = 0. ii. outside the shell y > R, log Ey= Oo” Amey 72 3. Electric field strength due to infinite line charge having linear charge density (4) coulomb/metre. ~ 4 2a “dare 7 4. Electric field strength near an infinite thin sheet of charge. a E= Dey 5. Electric field strength near a conductor a B= i) Electric field strength inside a conductor E = 0 Electric Potential: . WwW rie V= lim —=-[' Edi qo70dg =e Electric potential 1. due to a point charge, Ire 2. due to a charged conducting sphere or charged spherical shell of radius Ff. i. Inside. q Vinside = Smén ton = R) ii, Outside, : q, , Vout = > ut 4meg r (r R) Relation between electric field and potential pa LY tnumericatly) dr r - Work done in taking a charge ¢ from one point to another in electric field. W = q (V2 -Vy) joule where IV’; = potential at initial point, 2 = potential at final point. Work done in carrying a charge on equipotential surface is always zero. Electric Dipole: 1. Dipolemoment f= | 2I( Bi being the separation from - 4 to +4) 2. Torque on a dipole in uniform electric field x F wo Potential energy of dipole, U’ = — Pp. B= —pE cos 0 4. Work done in rotating the dipole in uniform electric field from orientation @; to @ is W=Ug — U)=pE (cas 0; = cos 03) Work done in rotating the dipole from equilibrium position @ = 0 to orientation @ is W=pk (1 - cos@) 5, Electric field due to a short dipole. i. at axial point Enns I 2p TAS Arey 73 ii. at equitorial point ~ i py BL aoe, i 6. Electric potential i. at axial point Vooe I Qp ans” Ae 42 ii, at equatorial point Vi=0 Capacitors 8. Dielectric constant — € _ Cineadium fg Cay 9. Capacitance of parallel plate capacitor ra 1.0 = in air d ay , 205 in medium of dielectric constant K d 3. When the space between the plates is partly filled with a dielectric of thicknescsc § then canacitance £9A / 1) d —t} 1-— é x} \ Ge 10. 1. Combination of Capacitors: i. Capacitors in series: Net capacitance C is given by Ppoiltil C2 C3 ii. In series charge is the same on all capacitors G1=q2=43 iii. Net potential difference V =I’; + V> + V3 2. Capacitors in Parallel: i. Net capacitance C = Cy + Co + C3 ii. Potential difference is same across all capacitors V)+V2+V3=V (same for all) iii, Charge g = q; + g2 + 43 2 41. Energy stored, U= ! cv2= & = ! av 2 2C 2 12. Electrostatic energy density, u,=/ I 27 . ) 3 ok” (in air) = 2 ek? (in medium) ’ 1 13. Induced charge ona dielectric, g =— a1 -1) \ 14, . 1 Total electric flux, =! E. ds= * net charge enclosed £0 by the surface. NUMERICALS NCERT Numericals Q. 1. A polythene piece rubbed with wool is found to have a negative charge of 3 * 1077 C. 1. Estimate the number of electrons transferred. Also state from which to which the transfer of electron took place. 2. Is there a transfer of mass from wool to polythene? Ans. Key idea: When two neutral bodies are rubbed together, electrons of ‘one body are transferred to the other. The body which gains. electrons is negatively charged and the body which loses electrons is positively charged 1. From quantisation of charge q = ne Hereg = 3x107? Cie =1.6x10719 © -. Number of electrons transferred, 3 -7 nat= 2% 10 py = 1875 x 1982 © 16x 19719 When polythene is rubbed with wool, the polythene becomes negatively charged and wool becomes positively charged. This implies that the electrons are transferred from wool to polythene. 2. As electrons have finite mass, the mass is transferred from wool to polythene AM = nx mg = 1.875 * 1012 x 9.1 x 10731 kg = 1.9 x 10718 kg Q. 2. What is the force between two small charged spheres having charges of 2 * 10-7 C and 3x 1077 C placed 30 cm apart in air ? Ans. Key idea: Two charged spheres at finite separation behave as point charge and the Coulomb's force of repulsion 1 gig _ qi92 dmeg 72 Here qi = 2% 107? C, qg=3 & 1077 C,r =30 ems 0.30 m (2x 10-7)(3 x 3 u?* x 1077) 6x 1073 F 10° N (0.30)? Q. 3. The electrostatic force on a small sphere of charge 0.4 pC due to another small sphere of charge — 0.8 pC in air is 0.2 N 41. What is the distance between the two spheres ? 2. What is the force on the second sphere due to the first ? Ans. Key idea: The electrostatic force between two charged spheres is given by Coulomb's law given by _ 1 4ife ~ 4rég 72 Here qix= 0.4 WC = 0.4 * 107°C, q2= — 0.8 pC = - 0.8 x 107°C = 08 x 107°C (magnitude), F = 0.2 N 1. As charges are of the opposite sign, the force between the charges magnitude is attractive (0.4 x 10-9) x (0.8 x 107%) “0.259 x 199%” } < “ 2 9x 10% x (04 x 107%) x (0.8 S2= x 174) 0.2 9x 16% ~ 1074 Q.4. Ans. 2. The force on second sphere due to first is = 0.2 N. Since | f; =lF ral. Ga=2nC Gg =—SpC Four point charges ¢4 = 2 wC, gg = — 5 uC, g¢ = 2 wC and qn = — 5 nC are located at the corners of a square ABCD of side 10 cm. What is the force on a charge of 1 wC placed at the centre of the sphere. Key idea: The coulomb's forces acting on a charged particle due to all other charges are added by vector method. Force on charge q7) =1 uC placed at centre O will be the vector sum of forces due to all the four charges 4.4. 73. qc and ¢qp . I 2 Clearly, OA =OB =OC =OD 55 V 102 + 10? _ V2 2 Force on gy =1 wC due to charge g4 = 2 pCis I 109A ( Fos)= dey (OAP 2 em=5V 2x 1072 along Oc (1 x 10-8(2 x 10-8) 9x 102 x = 3.6 Nalong Oc V2x 10-2)? Force on gy =J uC due to charge g, = 2 pC is I qoqe We along OA ( Fou) = 5 trey (OB? (1 x 10-82 x 10~%) 9x 109 x = 3.6 Nalong GA V2 10-22 Clearly, Foa + Foc =0 The force on gy = pC due to charge gy = —35 pC is I ode ( Fos) = tats along GB 4mey (OB) (1 x 10-65 x 10-8) - along OB=9.0 Nalong 9x 109 x« (5V210-2)2 SB The force on gy =/ wC due to charge gy = —5 pC is = qo = ( Fop) = tan, along GD trey (OD)? 1x 10-6 x 5 x 19-8 =9.0N along “OD 9 x 10° x V2x10-2F Clearly, For + Fon = 0 Therefore, net force on gy is Foat+ For* Foot Fan=¢ Foa + F= _~7 = +L Fou) eae paan ety ay OR. that is, the net force on charge qq is zero. Q. 5. Two point charges ¢4 = + 3C and gg = — 3 pC are located at points A and B, 20 cm apart in vacuum. 1. Find the electric field at the mid point of the line AB joining the two charges. 2. If a negative test charge of magnitude 1.5 * 10°? Cis placed at the centre, find the force experienced by the test charges. [CBSE Delhi 2003] Ans. — 0.1 mo —_——.1 m —— P—+E, meee Tg tonne E, 2 Qg= —3nC 1. The electric field strength at point P due to charges 4 and # is additive (away from positive charge and towards negative charge) -. Electric field strength at mid point due to charge ¢_; is TP 4a E,= ! Frey 92 3x 1076 =9 x 109x =2.7 x 10° NE~! along (0.10) AP Electric field strength at P due to charge 4, 1 4B 3x 19-8 _ IB gx 109 =2.7 x 10 direy 2 (0.40)? iC~/ along PB Net electric field at P Bo=By+ko=2.7 X10%+ 2.7 +109 =5.4 x109 NC~! along AB 2. Electric force on test charge gy placed at P F = qk = 1.5 * 107-9 x 5.4 x 106 =8.1 x ia-? Q. 6. A system has two charges qa = 2.5 x 1077 C and qg = - 2.5 * 1077 C located at points A= (0, 0, - 15 cm) and B= (0, 0, + 15 cm) respectively. What are the total charge and electric dipole moment of the system. Ans. Key idea: A dipole has two equal and opposite charges with dipole moment f'=9 2/ directed fram —q to —4. Given qa = 25X10 Co gy = 2.5 x 10-7 Total charge, g.7 + gg = 2.9x1077 C — 2.5K 107? C= 0. 21=AB =30 cem= 0.30 m peq. 2 Tdirected from —qtotg 5 x 1077 ©) (0.30 m) =7.5 x 107% Cm along B 3 =7.5 1078 Gm directed along negative z-axis. Q. 7. An electric dipole with a dipole moment 4 * 10-9 Cm is aligned at 30° with the direction of a uniform electric field of magnitude 5x 104NC71. Calculate the magnitude of the torque acting on the dipole. Ans. Key idea: A dipole placed in a uniform electric field, experiences a torque t= #F sin @ which tends to align the dipole parallel to the direction of field Torque = r= pF sin @ Here p = 4x07? Com, E =5x107 NCH!" 6 =30° Torque r=4x1079 x 5% 107 sin 30° g « 1 ~ =4x10-9 x 5 x 108 x 5 710 4Nm Q.8. E D 60" ASO 60 A B a=0.10m > A regular hexagon of side 0.10 m has a charge 5 uC at each of its vertices. Calculate the potential at the centre of the hexagon. Ans. Key idea: The potential due to similar charges is additive. Let O be the centre of the hexagon. In triangle OAB, all angles are 60°, so OA =OB = AB =a So, in a regular hexagon distance of each corner from centre is equal to the side of the hexagon ry =OA =OB =OC =OP =OF =OF =a = 0.10 m i The net potential at 0, V = 6x 4 Ireg a Here g =3uC =5 x 10-8 Cha = 0.10m 9 « 1g-® . 6 =6x 9x 10° x Fx = 2.7 x 10° volt O10 Q. 9. Two tiny spheres carrying charges 1.5 wC and 2.5 pC are located 30 cm apart. Find the potential and electric field 1. at the mid point of the line j g the two charges. 2. at a point 10 cm from this mid point in a plane normal to the line and passing through the mid point. Ans. A E,+#—-O—+E, B <+— 0.15 cm—> Electric field at P due toq;, 1 = Ey = u along P 3 Amey ry 15 x 197° =9 x 19%% 1° * 710 alongP (Q.18P Electric field at P due to qo 1 4 E> = along P RB ATE 19? 9, 25x 197% = =9 x 10% 7° along (0.18? Resolving £; and £5 along and normal to 4B. Net electric field along B a, Fy, = Fo cas @— Ey cos O «1 “ = | 28208 6 =1.5 x10 6 (333) (0-18)? 0-18 9x10? x1-0%107% (os (0-18) O18 Net electric field normal to AB, E, =(E> + Ey) sin 6 2-5x10"® +15x10-® | 0.40 (0-18)? 0-18 sags 40x10 8 26.2%109 NIC (0-18) 18 =(Ez ~E;) 008 0 =(Ey -E; }-23st08 NIC =9x10° =9x10" x Net electric field E = JE, = (2-3x 10°)? +(6-2% 10°)? = 6-6 10° N/C If « is the angle made by resultant field with 4B, then %y 62x 107 tangas = = 2.69 ex 23% 10 > a=tan ~! (2.69)=69.6° That resultant electric field at point P is 6.6 x10? N/C making an angle 69. 6° ta the line joining the charge 2.5 uC te 1.5 uC. Q. 10.4 cube of side b has a charge g at each of its vertices. Determine the potential and electric field due to this charge -array at the centre of the cube. Ans. a O is the centre of cube ABCDEFGH. Charge g is placed at each of eight corners of the cube. Electric Potential: Side of cube = b Length of each diagonal =Vi2 +0? + =V30 Distance of each corner from centre 0 Van 2 shalf the diagonal= I q Potential at O due to charge at each corner = = Arey (v'3Hi2) 1 24 trey V 3b .”. Net potential at O due to all 8 charges at corners of the cube vesx 24 1 164 y= 8x = trey Vay tte Vb Electric Field: The electric field at O due to charges at all corners of the cube is zero, since, electric fields due to charges at opposite corners such as 4 and //, G and D, B and £, F and C are equal and opposite Q. 11.An oil drop of 12 excess electrons is held stationary under a constant electric field of 2.55 * 104 NC“ in Millikan's oil drop experiment. The density of the oil is 1.26 g cm”, Estimate the radius of the drop (g = 9.81 ms~?, e = 1.60 X 1077? C). Ans. Key idea: In Millikan's oil drop experiment, the charged oil drop remains suspended (in equilibrium) when downward weight of drop. is balanced by upward electrostatic force and charge on drop q = ne, 1.0, qk = me > neE = mg « m F 4 If r is radius of oil drop, then mass m = — 47r?p 3 Here, n=12,¢=1-6x 107 © B= 255x104 NCW!) e=1-26gcm™ > =1.26x107 kg g=9-81lms~? o ay 12% 1-6% 1079 x 2-55 «10 4x3-14x 126x107 x 9-81 _[3x12*1-6% 2-55 x 1000)” “| dx314x1-26% 9-81 | 10-7 m =9-81«10-7 m=9-81« 10-4 mm Q. 12.If one of the two electrons of a hydrogen-molecule is removed, we get a hydrogen-molecular ion (H ; ). This ion consists of two protons and an electron. If at any instant the two protons be separated by a distance of 1.5 A and electrons by 1 A from each proton. Calculate the potential energy of system specify your choice of zero P.E. Ans. The choice of zero potential energy is when all charges are initially at infinite distance apart. The system of charges : 2 protons (each of charge + ¢) and an electron (of charge - ¢) is shown in figure The potential energy of system fe. e(-8 , ee) | Amtol tae fac Tac 4 / 14 1 | “neg” Law tae Fac Given: rag =1-5A =1-5% 1079 m rye =rg¢ =1A =107 m, e=16x10° °C. J 9x10" x eae? : i i 15x10-" 19° 19-9 =9x 2-56» 10 9 S=-30 72x 107? | Téxw« tonto inte el’ (keaning in mind 7 e¢l’ Ua -30.72 % 107'? py =— 39,2 eV 1.6 x 19-19 Thus, electrostatic potential energy of system U = — 30.72 x 10-19 joule or — 19.2 eV Q. 13.In a hydrogen atom, the electron and proton are bound at a distance of about 0.53 A, 1. Estimate the potential energy of the system in eV, taking the zero of potential energy at infinite separation of electron from proton. 2. What is the minimum work required to free the electron, given that its kinetic energy in the orbit is half the magnitude of potential energy obtained in (1). Ans. 1. Charge on proton g;= +/.6x10719 © Charge on electron go= — 1.6 10719 © Separation r = 0.53 4 = 053x107! m -. Potential energy of system U=Uatr—Uat Ig giz _y Ime y = 9 x 199 Eb LO? Ma 1.6 x 10719) as x 19-19 =- 43.47K 10-197] As I eV =1.6x107!9 J, we have _ 43.47 x 10°19 1.6x 10-1? 4D Kinatin anerny jo ahuave mncitiva on kinetin enerny at alartean U= eV = — 27.2eV 27.16 = 2? 4 13.6 eV 2 Total energy of electron = — 27,2 + 13.6 = -— 13.6 eV Minimum work required to free the electron = - Total energy of bound electron = 13.6 eV Q. 14.4 charge 8 mC is located at the origin. Calculate the work done in taking a small charge of -2x1079C from a point A(0, 0, 3 cm) to a point B(0, 4 cm, 0) via point C(O, 6 cm, 9 cm). Ans. iC (0, 6 cm, 9 cm) B (0, 4 cm, 0) Key idea: In electric field the work done in carrying a charge depends only on initial and final points and is independent of path. The points A, 6, C are shown in fig. Charge g=8 mC =8*1073 C is located at the origin O. Clearly, OA =ry =F em=3 x 1072 m OB =ry = 40m= 4 x 10-2 m Re aloctractatic field je cancarvative the wark dane ic indenendant of path. Hence, work done along path ACB (path 1) is same as work done along path 46 directly (path 2). By work-energy theorem, the work done is simply the change in electrostatic potential energy at two positions of charge qy (say) = — 2x10 °C. Work, IV’ = Potential energy of system when charge qq is at B — Potential energy of system when charge q_ is at A - 1 0 1 440 _ 1 aa (4-4) 4ney Tp ATG m4 Amey | r;, Substituting given values, we get W =9 «109 «(8x 10") x(-21079y} —+__-_1 4x10"? 3x107? ~1f 3-4 =-144 «10 ‘f +). 1-2 joule. Q. 15.Two charged conducting spheres of radii a and b are connected to each other by a conducting wire. What is the ratio of 1. charges on the spheres and 2. electric fields at the surfaces of the two spheres. Ans. 1. When conducting spheres are connected by a wire, the potential of each sphere will be the same. ie, V) =o If gp and q> are charges on them after connection, then Jog _ fa @ imag b . WW Ratio of charges |! = “ wf) mb That is, the ratio of charges on two spheres after their electrical contact is the same as the ratio of their radii. 2. Electric field strengths on the surfaces of two spheres 2 4néy 5 {using (7)] or Thus, the ratio of electric field strengths on their surfaces is equal to the inverse ratio of their radii Q. 16.Careful measurement of the electric field at the surface of a black-box indicates that the net outward flux through the surface of box is 8.0109 Nm?C7". 1. What is the net charge inside the box. 2. If the net outward flux through the surface of the box were zero, what can you conclude about the presence of charges inside the box. Ans. 1. Given electric flux @ = 8.0*10? Nm? C i From Gauss's theorem @ = 4g & ~. Charge enclosed, ¢ = sj ® =8.85x1/0~!? x 8.0 x 10? =70.8X1079 C= 70.8 2 2. If the net outward flux is zero, it indicates that the net charge enclosed in the blackbox is zero. The conclusion is either i. there is no charge inside the box or ii. there may be different types of charges in the box such that the algebraic sum of charges inside the box is zero. Q. 17.The electric field components in the fig. shown are FE, = a x!/2 : Ey = E, = 0, in which a = 800 N/ Cm!/2, Calculate 41. the electric flux through the cube 2. the charge within the cube. The side of cube a = 0./ m. [CBSE (Al) 2008C] Ans. 81+) @y— as, a] x ‘a a 1 Zz According to the question only the electric field has only X-component, ¥ and Z-components are zero; so electric flux diverging along Y and Z-axis is zero since EAS cas 90° = 0 The electric flux diverging along X-axis from faces (1) and (2) exists, The electric field at face (1) (where x = a)is 12 B, = ax? = @al’? The electric field at face (2) (where x = 2a) is Eg = ax!!? = @(2a)!/? The clectric fly thraiiqh facee (44 and (2) ara

Das könnte Ihnen auch gefallen